The Flat Earth Society

Flat Earth Discussion Boards => Flat Earth Theory => Topic started by: BlueMoon on March 10, 2016, 03:27:32 AM

Title: You wouldn't know how fast you're going
Post by: BlueMoon on March 10, 2016, 03:27:32 AM
Flat-earthers often say that it's unbelievable that we would be moving through space so fast. They say that the earth seems solid and stationary, and we would be able to feel if it was whizzing through space or spinning at 1000 mph. 

However, this is not the case.  Consider for a moment that you are riding in a bus.  While it is moving at a constant speed, you get up and move to the other side.  Why don't you get thrown to the back?  The reason is that you retain momentum, and you can only feel acceleration. 

Now, about that "1000 mph" statistic.  The equation for centripetal/centrifugal acceleration is a=v2/r.  The radius r is 6371 km, or 6371000 m.  THe velocity v is about 1000 mph, or 460 m/s.  So our function is 4602/6371000 which gives us...
.033 m/s2
For comparison, acceleration due to gravity at the poles is 9.83 m/s2.  You certainly wouldn't be flung off by that, but it has been measured. 

So why does the atmosphere stay with the earth's surface?  Well, it too has initial momentum.  This confines it to earth's reference frame. 
This initial momentum also explains the Coriolis effect.  Since the surface is spinning faster at the equator, and slower toward the poles, air that moves away from the equator is deflected to the east relative to the surface.  So, if you have an area of low pressure, air is drawn toward it, but air from the equator is deflected east, and air from the poles is deflected west.  That causes hurricanes to rotate clockwise in the southern hemisphere and counterclockwise in the northern hemisphere.  The deflection is measurable and consistent, and weather forecasters have to take it into account in their simulations.  It is also visible in the bands and storms of Jupiter. 

So what do you, the Flat Earth Society, have to say about that?  Can you find a better explanation that accounts for the weaker gravity at the equator and the Coriolis effect?  Good luck. 
Title: Re: You wouldn't know how fast you're going
Post by: rabinoz on March 10, 2016, 04:04:52 AM
So what do you, the Flat Earth Society, have to say about that?  Can you find a better explanation that accounts for the weaker gravity at the equator and the Coriolis effect?  Good luck.
I can't speak for TFES, but here is what they say:
Quote
The Coriolis Effect
Wind Currents
The Wind Currents are put into gradual motion by the attraction of the Northern and Southern Celestial Systems, which are grinding against each other as gears at the equator line.

Water Currents
As for water currents on a large scale; they're simply gradually put into motion by the winds. Water currents in the Northern Hemisphere will tend to rotate in one direction while currents in the Southern Hemisphere will tend to turn in another direction.
It does get just a bit hilarious with: grinding against each other as gears at the equator line. Doesn't seem to fit with highs and lows rotating in opposite directions.

Quote from: the Wiki
Celestial Gravitation
Celestial Gravitation is a part of some Flat Earth models which involve an attraction by all objects of mass on earth to the heavenly bodies. This is not the same as Gravity, since Celestial Gravitation does not imply an attraction between objects of mass on Earth. Celestial Gravitation accounts for tides and other gravimetric anomalies across the Earth's plane.

Mind it seems a bit ridiculous that the small sun and moon and tiny lights in the sky cause "Celestial Gravitation", but the almost infinitely more massive earth does not have any gravitation. But, who are we to doubt the Wiki?
Title: Re: You wouldn't know how fast you're going
Post by: Roundy on March 10, 2016, 05:11:26 AM
Mind it seems a bit ridiculous that the small sun and moon and tiny lights in the sky cause "Celestial Gravitation", but the almost infinitely more massive earth does not have any gravitation. But, who are we to doubt the Wiki?

What?  ??? Did someone here say that the Earth doesn't exhibit gravitation according to FET?  If so, I apologize that they misled you.
Title: Re: You wouldn't know how fast you're going
Post by: Woody on March 10, 2016, 05:22:28 AM
Mind it seems a bit ridiculous that the small sun and moon and tiny lights in the sky cause "Celestial Gravitation", but the almost infinitely more massive earth does not have any gravitation. But, who are we to doubt the Wiki?

What?  ??? Did someone here say that the Earth doesn't exhibit gravitation according to FET?  If so, I apologize that they misled you.

I think it depends on which site he is referring to.  I have read some post I do not remember where saying the force we think is gravity is a result of air pressure and outright deny the existence of gravity.  He may also be referring to UA saying the force we measure is the result of acceleration and not mass.  Which does not out right say gravity does not exist, but at least to me implies it.  If gravity existed and exerted the force as we are told it would make since the Earth would form into a spheroid.
Title: Re: You wouldn't know how fast you're going
Post by: Roundy on March 10, 2016, 05:35:13 AM
Mind it seems a bit ridiculous that the small sun and moon and tiny lights in the sky cause "Celestial Gravitation", but the almost infinitely more massive earth does not have any gravitation. But, who are we to doubt the Wiki?

What?  ??? Did someone here say that the Earth doesn't exhibit gravitation according to FET?  If so, I apologize that they misled you.

I think it depends on which site he is referring to.  I have read some post I do not remember where saying the force we think is gravity is a result of air pressure and outright deny the existence of gravity.  He may also be referring to UA saying the force we measure is the result of acceleration and not mass.  Which does not out right say gravity does not exist, but at least to me implies it.  If gravity existed and exerted the force as we are told it would make since the Earth would form into a spheroid.

Who said anything about gravity?  ???
Title: Re: You wouldn't know how fast you're going
Post by: BlueMoon on March 10, 2016, 05:39:39 AM
Mind it seems a bit ridiculous that the small sun and moon and tiny lights in the sky cause "Celestial Gravitation", but the almost infinitely more massive earth does not have any gravitation. But, who are we to doubt the Wiki?

What?  ??? Did someone here say that the Earth doesn't exhibit gravitation according to FET?  If so, I apologize that they misled you.

I think it depends on which site he is referring to.  I have read some post I do not remember where saying the force we think is gravity is a result of air pressure and outright deny the existence of gravity.  He may also be referring to UA saying the force we measure is the result of acceleration and not mass.  Which does not out right say gravity does not exist, but at least to me implies it.  If gravity existed and exerted the force as we are told it would make since the Earth would form into a spheroid.

Who said anything about gravity?  ???
Pretty sure I did in the OP.
Title: Re: You wouldn't know how fast you're going
Post by: Roundy on March 10, 2016, 05:41:11 AM
Mind it seems a bit ridiculous that the small sun and moon and tiny lights in the sky cause "Celestial Gravitation", but the almost infinitely more massive earth does not have any gravitation. But, who are we to doubt the Wiki?

What?  ??? Did someone here say that the Earth doesn't exhibit gravitation according to FET?  If so, I apologize that they misled you.

I think it depends on which site he is referring to.  I have read some post I do not remember where saying the force we think is gravity is a result of air pressure and outright deny the existence of gravity.  He may also be referring to UA saying the force we measure is the result of acceleration and not mass.  Which does not out right say gravity does not exist, but at least to me implies it.  If gravity existed and exerted the force as we are told it would make since the Earth would form into a spheroid.

Who said anything about gravity?  ???
Pretty sure I did in the OP.

We weren't talking about the OP, we were talking about a side comment by rabinoz that had little to do with the OP.
Title: Re: You wouldn't know how fast you're going
Post by: rabinoz on March 10, 2016, 06:05:28 AM
Mind it seems a bit ridiculous that the small sun and moon and tiny lights in the sky cause "Celestial Gravitation", but the almost infinitely more massive earth does not have any gravitation. But, who are we to doubt the Wiki?
What?  ??? Did someone here say that the Earth doesn't exhibit gravitation according to FET?  If so, I apologize that they misled you.
You might like to make up your own definitions, but the generally accepted definition seems to be: gravity is the gravitational field of the Earth (above its surface) and gravitation can describe the the gravitational field the general term.

Though I know we do often see "gravity" loosely applied to other objects as in the Moon's gravity.

But, what you say then gets back to how there can be any perceptible "Celestial Gravitation" on the earth's surface, yet none from the almost infinitely more massive earth. And if the Earth does exhibit gravitation according to FET cab we calculate its strength using Newton's Law of Gravitation, and if not how can it be calculated?
Title: Re: You wouldn't know how fast you're going
Post by: Roundy on March 10, 2016, 06:59:55 AM
Mind it seems a bit ridiculous that the small sun and moon and tiny lights in the sky cause "Celestial Gravitation", but the almost infinitely more massive earth does not have any gravitation. But, who are we to doubt the Wiki?
What?  ??? Did someone here say that the Earth doesn't exhibit gravitation according to FET?  If so, I apologize that they misled you.
You might like to make up your own definitions, but the generally accepted definition seems to be: gravity is the gravitational field of the Earth (above its surface) and gravitation can describe the the gravitational field the general term.

Though I know we do often see "gravity" loosely applied to other objects as in the Moon's gravity.


Gravitation is the tendency for some objects to be attracted to other objects.  "Gravity" is the generally accepted (and wrong) explanation for the tendency according to RET.  They are indeed two different things.  Celestial gravitation exists... celestial "gravity" does not.  The Earth exhibits gravitation (the most widely accepted explanation being universal acceleration), it does not exhibit "gravity".  I hope that clears up the confusion.
Title: Re: You wouldn't know how fast you're going
Post by: Panzerfaust on March 10, 2016, 07:03:50 AM
Mind it seems a bit ridiculous that the small sun and moon and tiny lights in the sky cause "Celestial Gravitation", but the almost infinitely more massive earth does not have any gravitation. But, who are we to doubt the Wiki?
What?  ??? Did someone here say that the Earth doesn't exhibit gravitation according to FET?  If so, I apologize that they misled you.
You might like to make up your own definitions, but the generally accepted definition seems to be: gravity is the gravitational field of the Earth (above its surface) and gravitation can describe the the gravitational field the general term.

Though I know we do often see "gravity" loosely applied to other objects as in the Moon's gravity.


Gravitation is the tendency for some objects to be attracted to other objects.  "Gravity" is the generally accepted (and wrong) explanation for the tendency according to RET.  They are indeed two different things.  Celestial gravitation exists... celestial "gravity" does not.  The Earth exhibits gravitation (the most widely accepted explanation being universal acceleration), it does not exhibit "gravity".  I hope that clears up the confusion.

I'm still a bit lost. How can attraction be selective?
Title: Re: You wouldn't know how fast you're going
Post by: BlueMoon on March 10, 2016, 07:13:49 AM
Mind it seems a bit ridiculous that the small sun and moon and tiny lights in the sky cause "Celestial Gravitation", but the almost infinitely more massive earth does not have any gravitation. But, who are we to doubt the Wiki?
What?  ??? Did someone here say that the Earth doesn't exhibit gravitation according to FET?  If so, I apologize that they misled you.
You might like to make up your own definitions, but the generally accepted definition seems to be: gravity is the gravitational field of the Earth (above its surface) and gravitation can describe the the gravitational field the general term.

Though I know we do often see "gravity" loosely applied to other objects as in the Moon's gravity.


Gravitation is the tendency for some objects to be attracted to other objects.  "Gravity" is the generally accepted (and wrong) explanation for the tendency according to RET.  They are indeed two different things.  Celestial gravitation exists... celestial "gravity" does not.  The Earth exhibits gravitation (the most widely accepted explanation being universal acceleration), it does not exhibit "gravity".  I hope that clears up the confusion.
Then why do the moon and other satellites follow the same laws when they travel around the earth, as the earth does when travelling around the sun?  And why do these same rules apply to the other planets and their moons?  Furthermore, why can we apply those laws to our own motion here on earth, as well as the tides? "Celestial gravitation" is a bogus explanation, and definitely doesn't pass Occam's Razor.
Title: Re: You wouldn't know how fast you're going
Post by: rabinoz on March 10, 2016, 08:50:05 AM
Mind it seems a bit ridiculous that the small sun and moon and tiny lights in the sky cause "Celestial Gravitation", but the almost infinitely more massive earth does not have any gravitation. But, who are we to doubt the Wiki?
What?  ??? Did someone here say that the Earth doesn't exhibit gravitation according to FET?  If so, I apologize that they misled you.
You might like to make up your own definitions, but the generally accepted definition seems to be: gravity is the gravitational field of the Earth (above its surface) and gravitation can describe the the gravitational field the general term.

Though I know we do often see "gravity" loosely applied to other objects as in the Moon's gravity.


Gravitation is the tendency for some objects to be attracted to other objects.  "Gravity" is the generally accepted (and wrong) explanation for the tendency according to RET.  They are indeed two different things.  Celestial gravitation exists... celestial "gravity" does not.  The Earth exhibits gravitation (the most widely accepted explanation being universal acceleration), it does not exhibit "gravity".  I hope that clears up the confusion.
Please come up some with actual evidence (that is not pure guesswork) for your selective "gravitation".
Gravitational attraction between quite a range of objects has been measured. Where do you have any evidence for your "Celestial gravitation".
I have asked many times, just what did Cavendish and the numerous others that performed similar experiments actually measure?
Go look up a bit more in: http://www.theflatearthsociety.org/forum/index.php?topic=66062.msg1765435#msg1765435 (http://www.theflatearthsociety.org/forum/index.php?topic=66062.msg1765435#msg1765435)

Maybe you have something to counter the numerous measurements of "G".

Usual Flat Earth logic! The Earth looks flat, so it must be flat! Then (literally in some cases) bend all other observations to fit.
Title: Re: You wouldn't know how fast you're going
Post by: TheTruthIsOnHere on March 10, 2016, 05:07:19 PM
Are you the same guy that guy exposed as a shill by wildheretic?
Title: Re: You wouldn't know how fast you're going
Post by: BlueMoon on March 10, 2016, 06:31:09 PM
Are you the same guy that guy exposed as a shill by wildheretic?
Depends on what you mean by "exposed as a shill."  If you mean "kicked off due to a misunderstanding," then yes.  He thought I was three different people simply because I posted some replies from the airport while I was traveling back from break, and then he was so busy patting himself on the back to realize that he was wrong.  Needless to say, he was an idiot.  It doesn't matter, though. 


But really, since this is off topic and personal, you should have PMed me. 
Title: Re: You wouldn't know how fast you're going
Post by: TheTruthIsOnHere on March 10, 2016, 07:06:32 PM
Can you tell me which principle makes our atmosphere basically an "encapsulated" entity? Ie: Why does the atmosphere not taper out in inverse squared fashion as described by gravitation. I think that is important to understand, as one would have to assume the atmosphere is the "vehicle" we're in that is moving otherwise, a bird should be able to fly 1,000 mph faster to the west.
Title: Re: You wouldn't know how fast you're going
Post by: BlueMoon on March 10, 2016, 10:00:55 PM
Can you tell me which principle makes our atmosphere basically an "encapsulated" entity? Ie: Why does the atmosphere not taper out in inverse squared fashion as described by gravitation. I think that is important to understand, as one would have to assume the atmosphere is the "vehicle" we're in that is moving otherwise, a bird should be able to fly 1,000 mph faster to the west.
The atmosphere doesn't taper out by the inverse square law because it is being held against the surface by gravity.  It extends away from the surface because of pressure. 
If I understand you correctly, the "vehicle" you're thinking of is the reference frame of the surface or atmosphere. 
Title: Re: You wouldn't know how fast you're going
Post by: TheTruthIsOnHere on March 10, 2016, 10:07:31 PM
Can you tell me which principle makes our atmosphere basically an "encapsulated" entity? Ie: Why does the atmosphere not taper out in inverse squared fashion as described by gravitation. I think that is important to understand, as one would have to assume the atmosphere is the "vehicle" we're in that is moving otherwise, a bird should be able to fly 1,000 mph faster to the west.
The atmosphere doesn't taper out by the inverse square law because it is being held against the surface by gravity.  It extends away from the surface because of pressure. 
If I understand you correctly, the "vehicle" you're thinking of is the reference frame of the surface or atmosphere.

Why is there a defined edge? Why does the ozone sit above our regular gaseous mixture that is a lot less dense than it? I'm just trying to understand how gravity holds our atmosphere and you have given yourself a lot of praise so you're surely qualified to teach me a thing or two.

You say that the surface of the earth has some kind of grip on air?
Title: Re: You wouldn't know how fast you're going
Post by: rabinoz on March 11, 2016, 04:45:54 AM
Can you tell me which principle makes our atmosphere basically an "encapsulated" entity? Ie: Why does the atmosphere not taper out in inverse squared fashion as described by gravitation. I think that is important to understand, as one would have to assume the atmosphere is the "vehicle" we're in that is moving otherwise, a bird should be able to fly 1,000 mph faster to the west.
The atmosphere doesn't taper out by the inverse square law because it is being held against the surface by gravity.  It extends away from the surface because of pressure. 
If I understand you correctly, the "vehicle" you're thinking of is the reference frame of the surface or atmosphere.

Why is there a defined edge? Why does the ozone sit above our regular gaseous mixture that is a lot less dense than it? I'm just trying to understand how gravity holds our atmosphere and you have given yourself a lot of praise so you're surely qualified to teach me a thing or two.

You say that the surface of the earth has some kind of grip on air?
I am sure you could read up on it as easily as anyone else can! But it seems that if you don't understand something you immediatley question its validity - or simply call it a fake. Most people (I hope) when they don't understand something they read up what they can on it, then make up their mind whether to question it or not.

"Why is there a defined edge? " - there is no "defined edge", the atmospheric pressure falls of roughly exponentially.
Now I'm no expert on ozone! But I would guess that ozone does not sit sit above our regular gaseous mixture, but the ozone layer is simply where most ozone is produced by ultra-violet light, which the ozone blocks (to some extent). The O3 will gradually sink, but being unstable will gradually revert to O2. See the diagram to the right.

The only ways the earth "grips" the atmosphere is gravity which keeps in on earth in the first place and drag which keeps it (generally[1]) moving at the same speed as the earth.
(https://upload.wikimedia.org/wikipedia/commons/thumb/e/ec/Atmospheric_ozone.svg/1162px-Atmospheric_ozone.svg.png)

[1] I say generally because there are other "forces" involved mainly such as pressure differences due largely to temperatures leading to north-south winds which are diverted by the Coriolis effect. There are then numerous things (such as mountain ranges)) that lead to more complicated wind patterns.
Title: Re: You wouldn't know how fast you're going
Post by: TheTruthIsOnHere on March 11, 2016, 04:11:54 PM
I was asking the NASA expert, rabinoz, thanks for volunteering though lol...

You still don't have an answer for how gravity seems to "taper off exponentially," as opposed to inverse squared distance from the surface of the earth. If your answer is pressure, density, and temperature, then why does gravity even need to be apart of the equation?

Title: Re: You wouldn't know how fast you're going
Post by: BlueMoon on March 11, 2016, 06:27:15 PM
I was asking the NASA expert, rabinoz, thanks for volunteering though lol...

You still don't have an answer for how gravity seems to "taper off exponentially," as opposed to inverse squared distance from the surface of the earth. If your answer is pressure, density, and temperature, then why does gravity even need to be apart of the equation?


I appreciate that you see me as the NASA expert, but rabinoz is correct. 


Gravity does follow the inverse square law, and, in general, extends to infinity. 


Atmosphere has no set boundary, but does not extend to infinity.  The reason for higher air pressure at the surface is not because of stronger gravity, but because of the weight of the column of air above it.  That's where gravity comes in. 


(http://nbviewer.jupyter.org/github/joshualim/ipython/blob/master/20150412_altitude_and_pressure/altitude_pressure.gif)
Title: Re: You wouldn't know how fast you're going
Post by: Hoppy on March 12, 2016, 01:50:54 AM
I was asking the NASA expert, rabinoz, thanks for volunteering though lol...

You still don't have an answer for how gravity seems to "taper off exponentially," as opposed to inverse squared distance from the surface of the earth. If your answer is pressure, density, and temperature, then why does gravity even need to be apart of the equation?
The made the whole thing up about gravity. It's kind of like saying magic is pulling things down.
Title: Re: You wouldn't know how fast you're going
Post by: BlueMoon on March 12, 2016, 03:53:33 AM
I was asking the NASA expert, rabinoz, thanks for volunteering though lol...

You still don't have an answer for how gravity seems to "taper off exponentially," as opposed to inverse squared distance from the surface of the earth. If your answer is pressure, density, and temperature, then why does gravity even need to be apart of the equation?
They made the whole thing up about gravity. It's kind of like saying magic is pulling things down.
Sort of, if by magic you mean the same exact magic that we've found applies to satellites, the moon, the earth, all the planets and their moons, and the sun, as well as the tides. And it sure as hell beats your "celestial gravitation" concept.  If it's not gravity accelerating things downward, then what is?  Why is there clearly higher atmospheric pressure at lower altitudes?  Bonus question: Why are there two high and low tides each day? 
Title: Re: You wouldn't know how fast you're going
Post by: rabinoz on March 12, 2016, 04:32:11 AM
I was asking the NASA expert, rabinoz, thanks for volunteering though lol...

You still don't have an answer for how gravity seems to "taper off exponentially," as opposed to inverse squared distance from the surface of the earth. If your answer is pressure, density, and temperature, then why does gravity even need to be apart of the equation?
The made the whole thing up about gravity. It's kind of like saying magic is pulling things down.

The whole thing up about the FE's Universal Acceleration. It's kind of like saying magic is pushing the whole Universe up at almost the speed of light!

But why is gravity any more magic than magnetic or electrostatic forces. The only real difference is that gravitation between typical objects is so miniscule that it is hard to (but not impossible) measure.
Title: Re: You wouldn't know how fast you're going
Post by: rabinoz on March 12, 2016, 04:48:42 AM
I was asking the NASA expert, rabinoz, thanks for volunteering though lol...

You still don't have an answer for how gravity seems to "taper off exponentially," as opposed to inverse squared distance from the surface of the earth. If your answer is pressure, density, and temperature, then why does gravity even need to be apart of the equation?

Who ever said I was "the NASA expert"! (Not I said the fly - as they say in the classics). I am no NASA expert. I do have a smattering of ideas - well one does after more decades than I would care to admit to - but for actual detail I look this up, as you could do just as easily!

There could be a good reason for "You still don't have an answer for how gravity seems to taper off exponentially".
That is because gravity DOES NOT taper off exponentially. If the earth is taken as having perfect spherical symmetry, then gravity falls off as the square of distance from the earth's centre of mass. The earth is not quite perfect sphere, but is extreme close. Also the nett gravitation will be affected by any other massive bodies in the vicinity (eg, the moon, but that is not very significant till we get about 10% of the way there)

I did say that the atmospheric pressure falls of approximately exponentially and that's a different kettle of fish altogther.
Title: Re: You wouldn't know how fast you're going
Post by: BlueMoon on March 12, 2016, 04:57:18 AM
I was asking the NASA expert, rabinoz, thanks for volunteering though lol...

You still don't have an answer for how gravity seems to "taper off exponentially," as opposed to inverse squared distance from the surface of the earth. If your answer is pressure, density, and temperature, then why does gravity even need to be apart of the equation?

Who ever said I was "the NASA expert"! (Not I said the fly - as they say in the classics). I am no NASA expert. I do have a smattering of ideas - well one does after more decades than I would care to admit to - but for actual detail I look this up, as you could do just as easily!

There could be a good reason for "You still don't have an answer for how gravity seems to taper off exponentially".
That is because gravity DOES NOT taper off exponentially. If the earth is taken as having perfect spherical symmetry, then gravity falls off as the square of distance from the earth's centre of mass. The earth is not quite perfect sphere, but is extreme close. Also the nett gravitation will be affected by any other massive bodies in the vicinity (eg, the moon, but that is not very significant till we get about 10% of the way there)

I did say that the atmospheric pressure falls of approximately exponentially and that's a different kettle of fish altogther.
Pretty sure he was talking about me, considering I'm the one that mentions NASA 3 times between my sig and my profile text  :P
You probably already knew that though.
Title: Re: You wouldn't know how fast you're going
Post by: TheTruthIsOnHere on March 12, 2016, 06:18:54 AM
Still no answer, just a bunch of misdirection and fluff. And flat earthers are the evasive ones. If gravity doesnt just stop, then why is there a distinct edge to our atmosphere? You know, the one that burns up objects the enter through it at 2500 degrees (thank god that doesn't effect manned space shuttles or we'd have some crispy cosmonauts)

Does the moon pull our atmosphere as it goes by? Or only water 100km further away? Is there any photo or video evidence of this?
Title: Re: You wouldn't know how fast you're going
Post by: BlueMoon on March 12, 2016, 07:18:37 AM
Still no answer, just a bunch of misdirection and fluff. And flat earthers are the evasive ones. If gravity doesnt just stop, then why is there a distinct edge to our atmosphere? You know, the one that burns up objects the enter through it at 2500 degrees (thank god that doesn't effect manned space shuttles or we'd have some crispy cosmonauts)

Does the moon pull our atmosphere as it goes by? Or only water 100km further away? Is there any photo or video evidence of this?


Actually, the moon does pull on the atmosphere.  Here (https://en.wikipedia.org/wiki/Atmospheric_tide)'s a link.  Unfortunately, I don't have any photo or video evidence for you since, y'know, it's the atmosphere. 


That "edge" you refer to doesn't actually exist.  Reentry doesn't have well defined borders, but you can think of it as the period of radio blackout (due to the ionization of air from shock heating. 
Reentry isn't an unsolvable problem.  However, if you try to make a ballistic reentry, you're gonna have a bad time.  That's why capsules and spaceplanes make a lifting reentry.  The capsules are designed to be off balance.  That allows them to control its roll during reentry in order to steer their way down, and it also allows them to spread out the deceleration, making for a safer, more comfortable, better controlled ride down.  The rest of the heat is managed by a heat shield using either ablative coating or advanced insulating tiles.  I, for one, find it fascinating. 


The ISS is still in the atmosphere, and orbital decay is a real issue for it (https://en.wikipedia.org/wiki/International_Space_Station#Orbit).  It reduces this by angling its solar panels to reduce drag at night, and corrects it by occasionally performing an orbital reboost. 


PS. I don't see what was wrong with my other answers.  To be honest, I really thought I nailed it.  I've laid out the correct reasons that the atmosphere doesn't have a set edge, and I even included a helpful graphic that I found in case you needed it.  What about that is fluff?  If you ask why the earth's atmosphere has a set edge, and then continue to insist that it has one despite repeated explanation to the contrary, can you really say you were looking for an answer? 
Title: Re: You wouldn't know how fast you're going
Post by: rabinoz on March 12, 2016, 07:48:40 AM
Still no answer, just a bunch of misdirection and fluff. And flat earthers are the evasive ones. If gravity doesnt just stop, then why is there a distinct edge to our atmosphere? You know, the one that burns up objects the enter through it at 2500 degrees (thank god that doesn't effect manned space shuttles or we'd have some crispy cosmonauts)

Does the moon pull our atmosphere as it goes by? Or only water 100km further away? Is there any photo or video evidence of this?
"Still no answer" to what?
There IS NO DISTINCT EDGE to our atmosphere!




There IS NO REGION THAT BURNS UP OBJECTS "that enter through it at 2500 degrees"! I imagine you are referring to the thermosphere. And to get you really concerned about "crispy cosmonauts" it is the region from about 100 km to 600 km - yes, you guessed it - right where the ISS orbits! I am sure this has been "put to bed" dozens of times, but here goes again:
You might also realise that the air density in this region is almost nothing! So, wht air there is might be very hot, but it is not dense enough to crisp a potato flake, let alone the ISS! take a gander at the picture on the right (from: http://www.geek.com/science/2200f-space-shuttle-heat-tiles-wont-burn-your-bare-hands-1559855/ (http://www.geek.com/science/2200f-space-shuttle-heat-tiles-wont-burn-your-bare-hands-1559855/)).
The "atmosphere" in the thermosphere is infinitely less dense (and poorer heat conductor) than that tile!


Why 2200°F space shuttle heat tiles won’t burn your bare hands
(http://www.geek.com/wp-content/uploads/2013/06/Tile.jpg)
"Does the moon pull our atmosphere as it goes by?" - by a miniscule amount!
And the moon (or sun) actually lift the water by only a small amount.
Have a look at the following from: https://tidesandcurrents.noaa.gov/education.html (https://tidesandcurrents.noaa.gov/education.html)
Quote
Types and Causes of Tidal Cycles - If the earth were a perfect sphere without large continents, all areas on the planet would experience two equally proportioned high and low tides every lunar day. The planet's large continents, however, block the westward passage of the tidal bulges as the earth rotates. Unable to move freely around the globe, these tides establish complex patterns within each ocean basin that often differ greatly from tidal patterns of adjacent ocean basins or other regions of the same ocean basin.
Quote
What Affects Tides? - The relative distances and positions of the sun, moon and earth all affect the size and magnitude of the earth's two tidal bulges. At a smaller scale, the magnitude of tides can be strongly influenced by the shape of the shoreline. When oceanic tidal bulges hit wide continental margins, the height of the tides can be magnified. Conversely, mid-oceanic islands not near continental margins typically experience very small tides of 1 meter or less.
The tides are caused by a relatively small wave moving around the earth "piling up" against continental margins. The very complicated sea-floor of the continental shelves lead to equally comlpex tide patterns in some area.

I really can't understand why you cannot look this stuff up for yourself. You seem to subscribe to the idea that if you can understand something about the globe, then it is evidence that the earth is flat. It is nothing of the sort! It simply that there are lots of things that you don't yet understand. There is nothing wrong with not understanding something. There are lots of things I don't understand. i will do some research - sometimes I can follow the reasoning, sometimes it is beyond me. Neither you nor I are the smartest people around and there will always be things we cannot yet find out, but we can try!
Title: Re: You wouldn't know how fast you're going
Post by: TheTruthIsOnHere on March 12, 2016, 07:11:27 PM
If there is no edge, then what's the difference between high altitude, low orbit, and high orbit?

SO have we never technically left the atmosphere? Besides supposedly in our space programs infancy in the Apollo missions?

If the ISS is in the atmosphere then why does the photos and videos supposedly taken from out the windows show a distinct edge, you know... The thing everyone talks about in regards to atmosphere?

And if you say that the gas particles in thermosphere are hot, what heats them up? Are the particles in the ISS immune from the suns radiation for whatever reason? I imagine it would be especially rough considering there isn't a lot of other molecules to transfer the heat to.

Give me a break.
Title: Re: You wouldn't know how fast you're going
Post by: rabinoz on March 12, 2016, 09:31:08 PM
If there is no edge, then what's the difference between high altitude, low orbit, and high orbit?

SO have we never technically left the atmosphere? Besides supposedly in our space programs infancy in the Apollo missions?

If the ISS is in the atmosphere then why does the photos and videos supposedly taken from out the windows show a distinct edge, you know... The thing everyone talks about in regards to atmosphere?

And if you say that the gas particles in thermosphere are hot, what heats them up? Are the particles in the ISS immune from the suns radiation for whatever reason? I imagine it would be especially rough considering there isn't a lot of other molecules to transfer the heat to.

Give me a break.
As I have stated elsewhere I have given up on answering innane questions from someone who will not open there own eyes and SEE!
Title: Re: You wouldn't know how fast you're going
Post by: BlueMoon on March 12, 2016, 09:43:32 PM
If there is no edge, then what's the difference between high altitude, low orbit, and high orbit?

SO have we never technically left the atmosphere? Besides supposedly in our space programs infancy in the Apollo missions?

If the ISS is in the atmosphere then why does the photos and videos supposedly taken from out the windows show a distinct edge, you know... The thing everyone talks about in regards to atmosphere?

And if you say that the gas particles in thermosphere are hot, what heats them up? Are the particles in the ISS immune from the suns radiation for whatever reason? I imagine it would be especially rough considering there isn't a lot of other molecules to transfer the heat to.

Give me a break.
You asked for it.  You don't get a break. 


The gas particles in the thermosphere are hot because they're exposed to the sun's radiation.  It doesn't take a whole lot of energy to heat up a single gas molecule.  However, it does take a lot of energy to heat up a solid space station.  The station is mostly covered in white insulation, which protects against the sun's rays.  It also has radiators to keep it cool.  The heat of the atmosphere isn't a problem because there are so few molecules. 


The photos of the "edge" from the ISS are probably of the ionosphere. 


(http://www.historyfuturenow.com/wp/wp-content/uploads/2012/09/Earths-atmosphere-and-lights-from-ISS.png)


We have technically left the atmosphere, when we went to the moon!  But since we haven't otherwise been beyond LEO, we've mostly within earth's atmosphere.  You could technically have an orbit at any height as long as it doesn't touch the ground, but the lower it is, the more drag and orbital decay becomes a problem. 
Title: Re: You wouldn't know how fast you're going
Post by: Roundy on March 13, 2016, 06:34:15 AM
Then why do the moon and other satellites follow the same laws when they travel around the earth, as the earth does when travelling around the sun?

They don't, in FET.  According to FET every celestial body is not traveling around the Earth, but rather the celestial hub (a fixed point above the north pole).  I'm sure you can see that it is not the same in any way as what RE has to say.

Quote
And why do these same rules apply to the other planets and their moons?  Furthermore, why can we apply those laws to our own motion here on earth, as well as the tides?

Because scientists wanted it that way.  It's all about how the math is interpreted; scientists choose to assume that gravitation works the same on Earth as it does in space, but their reasoning for choosing to do so is flawed.  It's really more faith-based than rational.
 
Title: Re: You wouldn't know how fast you're going
Post by: rabinoz on March 13, 2016, 12:04:41 PM
Then why do the moon and other satellites follow the same laws when they travel around the earth, as the earth does when travelling around the sun?
They don't, in FET.  According to FET every celestial body is not traveling around the Earth, but rather the celestial hub (a fixed point above the north pole).  I'm sure you can see that it is not the same in any way as what RE has to say.
Quote
And why do these same rules apply to the other planets and their moons?  Furthermore, why can we apply those laws to our own motion here on earth, as well as the tides?
Because scientists wanted it that way.  It's all about how the math is interpreted; scientists choose to assume that gravitation works the same on Earth as it does in space, but their reasoning for choosing to do so is flawed.  It's really more faith-based than rational.
 
You seem to claim "Celestial Gravitation" that even affects objects on the earth (supposedly the tides, etc), yet deny gravitation between the earth and objects on it. Surely the earth, objects on the earth and stellar objects are all the same kind of matter!

In any case gravitation between masses on the earth has been verified numerous times (measured).
I have asked many times, just what did Cavendish and the numerous others that performed similar experiments actually measure?

Some have accused Cavendish of knowing the answer beforehand and quessing the Universal Gravitational Constant "G". But, he never set out to measure "G", but to "weigh the earth" - find its density.
Newton before him, had no way of knowing this density so he assumed it would be about the same as the surface rocks - around 2,800 to 3,000 kg/m3.
This was all the information Cavendish had to start with. The result he got of about 5,500 kg/m3 surprised everybody,
but his results have been shown to be within about 1%. Not that bad for such a difficult experiment!
Of course once the mass of the earth was known "G" could easily be found, so he effectively measured the Universal Gravitational Constant "G".
Now before you dismiss Cavendish* (as so many Flat Earthers try to), just remember the value of G determined from the Cavendish experiment was within 1% of the currently accepted value.  You don't get that close by accident!

His result was verified in 1873 and there have been many modern version done to improve the accuracy. 
Most of the measurements were done using variations of the equipment used by Cavendish, though in at least one the equipment was evacuated to minimise interference.
The "atom interferometry" one uses "the minuscule gravitational tug between rubidium atoms and a 516-kilogram array of tungsten cylinders. The uncertainty in the latest measurement is 150 parts per million, or 0.015%" from the same source.

None of this pretends that gravitation is fully understood, but it appears to be a real phenomenon that causes an attractive force between two masses. (Pity we don't know how to reverse it yet!)

This table summarises the modern work:
(http://www.nature.com/polopoly_fs/7.17992.1403186108!/image/WEB_schlamminger.jpg_gen/derivatives/fullsize/WEB_schlamminger.jpg)
from: http://www.nature.com/news/quantum-method-closes-in-on-gravitational-constant-1.15427 (http://www.nature.com/news/quantum-method-closes-in-on-gravitational-constant-1.15427)

When one person does an experiment (like cold fusion or even detecting gravity waves) it might be looked on as interesting, but will not be taken too seriously until it can be shown to be repeatable. So the results of Cavendish's experiment could easily have been dismissed, had they not been verified numerous times.

No, it is a bit ridiculous to say that gravitation between objects on the earth has been de-bunked! And the term "gravity" in nothing more than the gravitational field due to the mass of the earth. In any case, you simply say it is non-existent, but never give any evidence - your denial of gravity is nothing more than a guess to prop up your hypothesis.


*  Some might argue that Miles Mathis has "debunked" Cavendish, but on reading his paper, I would not give much credence to it.  Mind you Miles Mathis seems to have had little to say on all the modern work, with better equipment and the means to avoid some of the sources of possible error.  In any case many of the "errors" Miles Mathis alludes to are simply constant masses in the vicinity, as no-one has questioned the additive property of gravity.
Another paper by Miles Mathis proves π = 4, and is not "dimensionless".  Interesting fellow, Miles Mathis!
Title: Re: You wouldn't know how fast you're going
Post by: Roundy on March 13, 2016, 05:22:41 PM

You seem to claim "Celestial Gravitation" that even affects objects on the earth (supposedly the tides, etc), yet deny gravitation between the earth and objects on it.

No I don't.
Title: Re: You wouldn't know how fast you're going
Post by: BlueMoon on March 13, 2016, 05:48:26 PM

You seem to claim "Celestial Gravitation" that even affects objects on the earth (supposedly the tides, etc), yet deny gravitation between the earth and objects on it.

No I don't.



Then can you please explain what your beliefs are?  Because they sure don't seem match up with other FEers. 


Also, I'm gonna have to point out that your answer is sub-minimum content. 
Title: Re: You wouldn't know how fast you're going
Post by: rabinoz on March 14, 2016, 01:17:15 AM

You seem to claim "Celestial Gravitation" that even affects objects on the earth (supposedly the tides, etc), yet deny gravitation between the earth and objects on it.

No I don't.

Quote from: the Wiki
Celestial Gravitation
Celestial Gravitation is a part of some Flat Earth models which involve an attraction by all objects of mass on earth to the heavenly bodies. This is not the same as Gravity, since Celestial Gravitation does not imply an attraction between objects of mass on Earth. Celestial Gravitation accounts for tides and other gravimetric anomalies across the Earth's plane.

Well, then what about some answers! I asked the following:
Quote from: rabinoz
In any case gravitation between masses on the earth has been verified numerous times (measured).
I have asked many times, just what did Cavendish and the numerous others that performed similar experiments actually measure?
We are constantly told "gravity" (which in common usage is nothing more than the "gravitational field due to the earth") is a "fake".
We are constantly told to "look up the Wiki"! We do that and then you say you don't believe that either!

While you are at it please explain the measured variations in gravity on the earth's surface (this has been well known for hundreds or years) and the fact that we get TWO tides a day in most places.

I would say that if Flat Earthers have no answer to what Cavendish and the numerous others that performed similar experiments actually measured, then just accept that gravity is real.
Title: Re: You wouldn't know how fast you're going
Post by: Roundy on March 14, 2016, 05:38:01 AM
Gravitation exists on the Earth.  You throw a ball up, it comes back down.  Obviously that is a form of gravitation.  I feel like I've only recently mentioned this to you, but you should look up the universal accelerator in the wiki.
Title: Re: You wouldn't know how fast you're going
Post by: rabinoz on March 14, 2016, 11:47:11 AM
Gravitation exists on the Earth.  You throw a ball up, it comes back down.  Obviously that is a form of gravitation.  I feel like I've only recently mentioned this to you, but you should look up the universal accelerator in the wiki.
You (along with I believe every FEer) have not yet answered my question.
Quote
In any case gravitation between masses on the earth has been verified numerous times (measured).
I have asked many times, just what did Cavendish and the numerous others that performed similar experiments actually measure?

I had hoped not to repeat all this again, but I guess I have to!

So many dismiss gravitation, but just what did Cavendish and the numerous others that performed similar experiments actually measure?

Some have accused Cavendish of knowing the answer beforehand and quessing the Universal Gravitational Constant "G". But, he never set out to measure "G", but to "weigh the earth" - find its density.
Newton before him, had no way of knowing this density so he assumed it would be about the same as the surface rocks - around 2,800 to 3,000 kg/m3.
This was all the information Cavendish had to start with. The result of about 5,500 kg/m3 surprised everybody,
but his results have been shown to be within about 1%. Not that bad for such a difficult experiment!
Of course once the mass of the earth was known "G" could easily be found, so he effectively measured the Universal Gravitational Constant "G".
Now before you dismiss Cavendish[1] (as so many Flat Earthers try to), just remember the value of "G" determined from the Cavendish experiment was within 1% of the currently accepted value.  You don't get that close by accident! His result was verified in 1873 and there have been many modern version done to improve the accuracy. 

There have been numerous versions of the Cavendish experiment performed since then.
His result was verified in 1873 and there have been many modern version done to improve the accuracy. 
Most of the measurements were done using variations of the equipment used by Cavendish, though in at least one the equipment was evacuated to minimise interference.
The "atom interferometry" one uses "the minuscule gravitational tug between rubidium atoms and a 516-kilogram array of tungsten cylinders. The uncertainty in the latest measurement is 150 parts per million, or 0.015%" from the same source.

None of this pretends that gravitation is fully understood, but it appears to be a real phenomenon that causes an attractive force between two masses. (Pity we don't know how to reverse it yet!)

This table summarises the modern work:
(http://www.nature.com/polopoly_fs/7.17992.1403186108!/image/WEB_schlamminger.jpg_gen/derivatives/fullsize/WEB_schlamminger.jpg)
from: http://www.nature.com/news/quantum-method-closes-in-on-gravitational-constant-1.15427 (http://www.nature.com/news/quantum-method-closes-in-on-gravitational-constant-1.15427)

When one person does an experiment (like cold fusion or even detecting gravity waves) it might be looked on as interesting, but will not be taken too seriously until it can be shown to be repeatable. So the results of Cavendish's experiment could easily have been dismissed, had they not been verified numerous times.

[1]  Some might argue that Miles Mathis has "debunked" Cavendish, but on reading his paper, I would not give much credence to it.  Mind you Miles Mathis seems to have had little to say on all the modern work, with better equipment and the means to avoid some of the sources of possible error.  In any case many of the "errors" Miles Mathis alludes to are simply constant masses in the vicinity, as no-one has questioned the additive property of gravity.
Another paper by Miles Mathis proves π = 4, and is not "dimensionless".  Interesting fellow, Miles Mathis!
Title: Re: You wouldn't know how fast you're going
Post by: Roundy on March 15, 2016, 02:25:31 AM
Gravitation exists on the Earth.  You throw a ball up, it comes back down.  Obviously that is a form of gravitation.  I feel like I've only recently mentioned this to you, but you should look up the universal accelerator in the wiki.
You (along with I believe every FEer) have not yet answered my question.
Quote
In any case gravitation between masses on the earth has been verified numerous times (measured).
I have asked many times, just what did Cavendish and the numerous others that performed similar experiments actually measure?

I had hoped not to repeat all this again, but I guess I have to!

So many dismiss gravitation, but just what did Cavendish and the numerous others that performed similar experiments actually measure?

Some have accused Cavendish of knowing the answer beforehand and quessing the Universal Gravitational Constant "G". But, he never set out to measure "G", but to "weigh the earth" - find its density.
Newton before him, had no way of knowing this density so he assumed it would be about the same as the surface rocks - around 2,800 to 3,000 kg/m3.
This was all the information Cavendish had to start with. The result of about 5,500 kg/m3 surprised everybody,
but his results have been shown to be within about 1%. Not that bad for such a difficult experiment!
Of course once the mass of the earth was known "G" could easily be found, so he effectively measured the Universal Gravitational Constant "G".
Now before you dismiss Cavendish[1] (as so many Flat Earthers try to), just remember the value of "G" determined from the Cavendish experiment was within 1% of the currently accepted value.  You don't get that close by accident! His result was verified in 1873 and there have been many modern version done to improve the accuracy. 

There have been numerous versions of the Cavendish experiment performed since then.
His result was verified in 1873 and there have been many modern version done to improve the accuracy. 
Most of the measurements were done using variations of the equipment used by Cavendish, though in at least one the equipment was evacuated to minimise interference.
The "atom interferometry" one uses "the minuscule gravitational tug between rubidium atoms and a 516-kilogram array of tungsten cylinders. The uncertainty in the latest measurement is 150 parts per million, or 0.015%" from the same source.

None of this pretends that gravitation is fully understood, but it appears to be a real phenomenon that causes an attractive force between two masses. (Pity we don't know how to reverse it yet!)

This table summarises the modern work:
(http://www.nature.com/polopoly_fs/7.17992.1403186108!/image/WEB_schlamminger.jpg_gen/derivatives/fullsize/WEB_schlamminger.jpg)
from: http://www.nature.com/news/quantum-method-closes-in-on-gravitational-constant-1.15427 (http://www.nature.com/news/quantum-method-closes-in-on-gravitational-constant-1.15427)

When one person does an experiment (like cold fusion or even detecting gravity waves) it might be looked on as interesting, but will not be taken too seriously until it can be shown to be repeatable. So the results of Cavendish's experiment could easily have been dismissed, had they not been verified numerous times.

[1]  Some might argue that Miles Mathis has "debunked" Cavendish, but on reading his paper, I would not give much credence to it.  Mind you Miles Mathis seems to have had little to say on all the modern work, with better equipment and the means to avoid some of the sources of possible error.  In any case many of the "errors" Miles Mathis alludes to are simply constant masses in the vicinity, as no-one has questioned the additive property of gravity.
Another paper by Miles Mathis proves π = 4, and is not "dimensionless".  Interesting fellow, Miles Mathis!

Well, I apologize, after seeing that the rest of your post was based on a faulty proposition I decided to skip it, as your posts sometimes give me a headache (this time for a change at least the meandering was somewhat on topic), but as it happens, I don't know what's being observed in the Cavendish Experiment.  Obviously something is causing something to react somehow, but I do think that Miles Mathis makes some excellent points that can't be dismissed out-of-hand, however eccentric he might seem (Sir Isaac Newton was an alchemist yet he's revered as something of a God to you people, after all) and we must be cognizant of the fact that there's no reason to assume that the theory behind the Cavendish Experiment isn't flawed, or misunderstood.

This is why an open-minded approach to FE research is so important.  Rather than deny something that has been observed many, many times by people of all walks of life, we should be pondering this fascinating experiment, and striving to understand it.  To me, that's what the modern Flat Earth Society is all about, or at least should be.

Whatever the case may be, the Cavendish Experiment does not prove that the Earth is not flat.
Title: Re: You wouldn't know how fast you're going
Post by: rabinoz on March 15, 2016, 08:59:12 AM
Well, I apologize, after seeing that the rest of your post was based on a faulty proposition I decided to skip it, as your posts sometimes give me a headache (this time for a change at least the meandering was somewhat on topic), but as it happens, I don't know what's being observed in the Cavendish Experiment.  Obviously something is causing something to react somehow, but I do think that Miles Mathis makes some excellent points that can't be dismissed out-of-hand, however eccentric he might seem (Sir Isaac Newton was an alchemist yet he's revered as something of a God to you people, after all) and we must be cognizant of the fact that there's no reason to assume that the theory behind the Cavendish Experiment isn't flawed, or misunderstood.

This is why an open-minded approach to FE research is so important.  Rather than deny something that has been observed many, many times by people of all walks of life, we should be pondering this fascinating experiment, and striving to understand it.  To me, that's what the modern Flat Earth Society is all about, or at least should be.

Whatever the case may be, the Cavendish Experiment does not prove that the Earth is not flat.
Well, the TFES certainly seems to regard Rowbotham as a prophet if not a "god", but no-one regards Newton or any other others as "gods".
Copernicus, Brahe, Kepler,  Galileo, Newton, et al did not invent anything or make up anything. They simply discovered what was there. If they didn't someone else would have!

And no, "the Cavendish Experiment does not prove that the Earth is not flat", but it does demonstrate that there is an attractive force between objects with mass. Not only that, but the force is given by:
F = G x m1 x m2 / D2
Hence the Flat Earth must have its own gravity! It would not be the same as on the Globe and would not be simply downward.

It would have a "hubward" (sorry discworld terminology creeps in) component. That is your guru Rowbotham has to try to deny gravity.

I have sympathy for your headaches, but there are so many other areas where your hypothesis simply does not match the real earth.
Title: Re: You wouldn't know how fast you're going
Post by: UnionsOfSolarSystemPlanet on March 15, 2016, 10:37:50 AM
Gravitation exists on the Earth.  You throw a ball up, it comes back down.  Obviously that is a form of gravitation.  I feel like I've only recently mentioned this to you, but you should look up the universal accelerator in the wiki.
You (along with I believe every FEer) have not yet answered my question.
Quote
In any case gravitation between masses on the earth has been verified numerous times (measured).
I have asked many times, just what did Cavendish and the numerous others that performed similar experiments actually measure?

I had hoped not to repeat all this again, but I guess I have to!

So many dismiss gravitation, but just what did Cavendish and the numerous others that performed similar experiments actually measure?

Some have accused Cavendish of knowing the answer beforehand and quessing the Universal Gravitational Constant "G". But, he never set out to measure "G", but to "weigh the earth" - find its density.
Newton before him, had no way of knowing this density so he assumed it would be about the same as the surface rocks - around 2,800 to 3,000 kg/m3.
This was all the information Cavendish had to start with. The result of about 5,500 kg/m3 surprised everybody,
but his results have been shown to be within about 1%. Not that bad for such a difficult experiment!
Of course once the mass of the earth was known "G" could easily be found, so he effectively measured the Universal Gravitational Constant "G".
Now before you dismiss Cavendish[1] (as so many Flat Earthers try to), just remember the value of "G" determined from the Cavendish experiment was within 1% of the currently accepted value.  You don't get that close by accident! His result was verified in 1873 and there have been many modern version done to improve the accuracy. 

There have been numerous versions of the Cavendish experiment performed since then.
His result was verified in 1873 and there have been many modern version done to improve the accuracy. 
Most of the measurements were done using variations of the equipment used by Cavendish, though in at least one the equipment was evacuated to minimise interference.
The "atom interferometry" one uses "the minuscule gravitational tug between rubidium atoms and a 516-kilogram array of tungsten cylinders. The uncertainty in the latest measurement is 150 parts per million, or 0.015%" from the same source.

None of this pretends that gravitation is fully understood, but it appears to be a real phenomenon that causes an attractive force between two masses. (Pity we don't know how to reverse it yet!)

This table summarises the modern work:
(http://www.nature.com/polopoly_fs/7.17992.1403186108!/image/WEB_schlamminger.jpg_gen/derivatives/fullsize/WEB_schlamminger.jpg)
from: http://www.nature.com/news/quantum-method-closes-in-on-gravitational-constant-1.15427 (http://www.nature.com/news/quantum-method-closes-in-on-gravitational-constant-1.15427)

When one person does an experiment (like cold fusion or even detecting gravity waves) it might be looked on as interesting, but will not be taken too seriously until it can be shown to be repeatable. So the results of Cavendish's experiment could easily have been dismissed, had they not been verified numerous times.

[1]  Some might argue that Miles Mathis has "debunked" Cavendish, but on reading his paper, I would not give much credence to it.  Mind you Miles Mathis seems to have had little to say on all the modern work, with better equipment and the means to avoid some of the sources of possible error.  In any case many of the "errors" Miles Mathis alludes to are simply constant masses in the vicinity, as no-one has questioned the additive property of gravity.
Another paper by Miles Mathis proves π = 4, and is not "dimensionless".  Interesting fellow, Miles Mathis!

Well, I apologize, after seeing that the rest of your post was based on a faulty proposition I decided to skip it, as your posts sometimes give me a headache (this time for a change at least the meandering was somewhat on topic), but as it happens, I don't know what's being observed in the Cavendish Experiment.  Obviously something is causing something to react somehow, but I do think that Miles Mathis makes some excellent points that can't be dismissed out-of-hand, however eccentric he might seem (Sir Isaac Newton was an alchemist yet he's revered as something of a God to you people, after all) and we must be cognizant of the fact that there's no reason to assume that the theory behind the Cavendish Experiment isn't flawed, or misunderstood.

This is why an open-minded approach to FE research is so important.  Rather than deny something that has been observed many, many times by people of all walks of life, we should be pondering this fascinating experiment, and striving to understand it.  To me, that's what the modern Flat Earth Society is all about, or at least should be.

Whatever the case may be, the Cavendish Experiment does not prove that the Earth is not flat.

So you're basically saying "I don't get what the Cavendish Experiment is, therefor it doesn't prove the Earth is not flat"

Cavendish was trying to measure the density of Earth, which since the experiment was a success, it prove that the gravity you claimed to believe, is indeed a result of attraction of Earth's mass, which would mean if the Earth is flat gravity would pull it back to a sphere. For the Earth to be flat, the gravity we experience on the ground must not result from Earth's mass, which would require the Cavendish Experiment and all of it's modern attempts to be a hoax.
Title: Re: You wouldn't know how fast you're going
Post by: TheTruthIsOnHere on March 15, 2016, 03:39:12 PM
Well, the TFES certainly seems to regard Rowbotham as a prophet if not a "god", but no-one regards Newton or any other others as "gods".
Copernicus, Brahe, Kepler,  Galileo, Newton, et al did not invent anything or make up anything. They simply discovered what was there. If they didn't someone else would have!

You know who did make up something? Einstein (or he plagiarized it, some say)

Relativity is the patchwork that covered up the holes in Newton, Kepler, and other natural philosopher's theories and "thought experiments."

Something not adding up? Dark Matter. Light bending? Black Holes. Want to really fuck with their heads? Let's create the space time continuum.

Fantasy bullshit made up to distract you from the flaws inherent in our concepts of the Universe. I'm beginning to think Einstein was contracted to create a concept to complex to question, in order to continue the status quo.
Title: Re: You wouldn't know how fast you're going
Post by: BlueMoon on March 15, 2016, 05:21:32 PM
Well, the TFES certainly seems to regard Rowbotham as a prophet if not a "god", but no-one regards Newton or any other others as "gods".
Copernicus, Brahe, Kepler,  Galileo, Newton, et al did not invent anything or make up anything. They simply discovered what was there. If they didn't someone else would have!

You know who did make up something? Einstein (or he plagiarized it, some say)

Relativity is the patchwork that covered up the holes in Newton, Kepler, and other natural philosopher's theories and "thought experiments."

Something not adding up? Dark Matter. Light bending? Black Holes. Want to really fuck with their heads? Let's create the space time continuum.

Fantasy bullshit made up to distract you from the flaws inherent in our concepts of the Universe. I'm beginning to think Einstein was contracted to create a concept to complex to question, in order to continue the status quo.
It sure beats your aether.  That's just fantasy bullshit to explain away the evidence that the earth is round.  At least our explanations are mathematically defined and tested, and maintain a level of internal consistency. 
Title: Re: You wouldn't know how fast you're going
Post by: TheTruthIsOnHere on March 15, 2016, 05:36:27 PM
Well, the TFES certainly seems to regard Rowbotham as a prophet if not a "god", but no-one regards Newton or any other others as "gods".
Copernicus, Brahe, Kepler,  Galileo, Newton, et al did not invent anything or make up anything. They simply discovered what was there. If they didn't someone else would have!

You know who did make up something? Einstein (or he plagiarized it, some say)

Relativity is the patchwork that covered up the holes in Newton, Kepler, and other natural philosopher's theories and "thought experiments."

Something not adding up? Dark Matter. Light bending? Black Holes. Want to really fuck with their heads? Let's create the space time continuum.

Fantasy bullshit made up to distract you from the flaws inherent in our concepts of the Universe. I'm beginning to think Einstein was contracted to create a concept to complex to question, in order to continue the status quo.
It sure beats your aether.  That's just fantasy bullshit to explain away the evidence that the earth is round.  At least our explanations are mathematically defined and tested, and maintain a level of internal consistency.

You've never once seen me say anything at all about aether.

And if I was to say anything about it, I would mention to you how aether theory had nothing to do with the shape of the Earth. Tesla was a huge proponent of the Aether, do you think he thought the Earth was flat?
Title: Re: You wouldn't know how fast you're going
Post by: BlueMoon on March 15, 2016, 06:29:26 PM
Well, the TFES certainly seems to regard Rowbotham as a prophet if not a "god", but no-one regards Newton or any other others as "gods".
Copernicus, Brahe, Kepler,  Galileo, Newton, et al did not invent anything or make up anything. They simply discovered what was there. If they didn't someone else would have!

You know who did make up something? Einstein (or he plagiarized it, some say)

Relativity is the patchwork that covered up the holes in Newton, Kepler, and other natural philosopher's theories and "thought experiments."

Something not adding up? Dark Matter. Light bending? Black Holes. Want to really fuck with their heads? Let's create the space time continuum.

Fantasy bullshit made up to distract you from the flaws inherent in our concepts of the Universe. I'm beginning to think Einstein was contracted to create a concept to complex to question, in order to continue the status quo.
It sure beats your aether.  That's just fantasy bullshit to explain away the evidence that the earth is round.  At least our explanations are mathematically defined and tested, and maintain a level of internal consistency.

You've never once seen me say anything at all about aether.

And if I was to say anything about it, I would mention to you how aether theory had nothing to do with the shape of the Earth. Tesla was a huge proponent of the Aether, do you think he thought the Earth was flat?
Aether has been disproven since Tesla's time. 
Gravity, relativity and quantum mechanics have been proven. 
Now, I remember that you avoid mentioning aether or claiming that the earth is flat.  But by disavowing relativity, you're saying that there is a better explanation, and on these forums it's always aether. 
Title: Re: You wouldn't know how fast you're going
Post by: TheTruthIsOnHere on March 15, 2016, 07:27:26 PM
Well, the TFES certainly seems to regard Rowbotham as a prophet if not a "god", but no-one regards Newton or any other others as "gods".
Copernicus, Brahe, Kepler,  Galileo, Newton, et al did not invent anything or make up anything. They simply discovered what was there. If they didn't someone else would have!

You know who did make up something? Einstein (or he plagiarized it, some say)

Relativity is the patchwork that covered up the holes in Newton, Kepler, and other natural philosopher's theories and "thought experiments."

Something not adding up? Dark Matter. Light bending? Black Holes. Want to really fuck with their heads? Let's create the space time continuum.

Fantasy bullshit made up to distract you from the flaws inherent in our concepts of the Universe. I'm beginning to think Einstein was contracted to create a concept to complex to question, in order to continue the status quo.
It sure beats your aether.  That's just fantasy bullshit to explain away the evidence that the earth is round.  At least our explanations are mathematically defined and tested, and maintain a level of internal consistency.

You've never once seen me say anything at all about aether.

And if I was to say anything about it, I would mention to you how aether theory had nothing to do with the shape of the Earth. Tesla was a huge proponent of the Aether, do you think he thought the Earth was flat?
Aether has been disproven since Tesla's time. 
Gravity, relativity and quantum mechanics have been proven. 
Now, I remember that you avoid mentioning aether or claiming that the earth is flat.  But by disavowing relativity, you're saying that there is a better explanation, and on these forums it's always aether.
Incredibly flawed logic, to assume that just because I'm on this forum you can fill in my unstated opinions on a subject with that of a select few others. But since you're the expert, please in your own words explain general relativity, and in plain english also explain what you imagine aether to be.

Also, please explain how something can be disproved? Things can just fail to be proven, but to disprove something, that's entirely more difficult to do. ie: existence of God. It's not proven, but it's also impossible to disprove.
Title: Re: You wouldn't know how fast you're going
Post by: rabinoz on March 15, 2016, 10:37:08 PM
Well, the TFES certainly seems to regard Rowbotham as a prophet if not a "god", but no-one regards Newton or any other others as "gods".
Copernicus, Brahe, Kepler,  Galileo, Newton, et al did not invent anything or make up anything. They simply discovered what was there. If they didn't someone else would have!

You know who did make up something? Einstein (or he plagiarized it, some say)

Relativity is the patchwork that covered up the holes in Newton, Kepler, and other natural philosopher's theories and "thought experiments."

Something not adding up? Dark Matter. Light bending? Black Holes. Want to really fuck with their heads? Let's create the space time continuum.

Fantasy bullshit made up to distract you from the flaws inherent in our concepts of the Universe. I'm beginning to think Einstein was contracted to create a concept to complex to question, in order to continue the status quo.
I have never yet seen anything that indicates the you have the slightest knowledge of the Flat Earth you purport to support.
You talk about "Light bending?" On your flat earth model (you do have one?)

You worry about "dark matter", but the Heliocentric Globe does NOT rely on dark matter one little bit! Yes it is hypothesized to explain galaxies "holding together"!

On the other hand the accepted Flat Earth depends on "Dark Energy" to power its Universal Acceleration that provides its "simulated gravity". This hypothesis provides no explanation for measured variations in "g".

While the Globe uses well proven gravitation[2] to provide genuine gravity that varies across the earth as explained by rotation and differing polar and equatorial radii and variation with altitude!

The "the flaws inherent in our concepts of the Universe" are simply flaws in your understanding of it. Don't blame Newton, Einstein or anyone else for your failings - go learn a bit about the Globe and more particularly about what alternatives there are! I do not see ANY that explain observations.
Yes, I know you won't consider any of this, but maybe someone else will see the fallacy of the Flat Earth model.

[1] Except right at the poles - they BOTH get 24 hour sunlight on the equinoxes - explain THAT with you Flat Earth sun!
[2] See: "Henry Cavendish proved Newton's Gravitation" http://www.theflatearthsociety.org/forum/index.php?topic=66174.0 (http://www.theflatearthsociety.org/forum/index.php?topic=66174.0)[/list]
Title: Re: You wouldn't know how fast you're going
Post by: Unsure101 on March 16, 2016, 11:34:07 AM
Gravitation exists on the Earth.  You throw a ball up, it comes back down.  Obviously that is a form of gravitation.  I feel like I've only recently mentioned this to you, but you should look up the universal accelerator in the wiki.
Sorry to join the party late, but if the FE is accelerating upwards, why haven't we crashed into the sun and Moon yet? They're only a few miles up yes?
You throw a ball up, it falls back to the earth. You launch a weather balloon, it falls back to earth, yet the sun and Moon don't. What magic holds them up, but everything else falls?
Title: Re: You wouldn't know how fast you're going
Post by: TheTruthIsOnHere on March 16, 2016, 12:42:27 PM
You know who did make up something? Einstein (or he plagiarized it, some say)

Relativity is the patchwork that covered up the holes in Newton, Kepler, and other natural philosopher's theories and "thought experiments."

Something not adding up? Dark Matter. Light bending? Black Holes. Want to really fuck with their heads? Let's create the space time continuum.

Fantasy bullshit made up to distract you from the flaws inherent in our concepts of the Universe. I'm beginning to think Einstein was contracted to create a concept to complex to question, in order to continue the status quo.
I have never yet seen anything that indicates the you have the slightest knowledge of the Flat Earth you purport to support.
You talk about "Light bending?" On your flat earth model (you do have one?)
  • at the time the sun is setting it is still 3,000 miles above the earth (about 20° if you want an angle!), yet we see the sun appearing to set behind the horizon! - BENDY LIGHT?
  • An equinox is coming up (Mar 20). On that date the sun will be able to be seen to rise due east (90°) and set due west - everywhere![1] Yet on the accepted (?) flat earth, with the sun circling over the equator the sun would rise even north of NW (at about 36°) - BENDY LIGHT?

You worry about "dark matter", but the Heliocentric Globe does NOT rely on dark matter one little bit! Yes it is hypothesized to explain galaxies "holding together"!

On the other hand the accepted Flat Earth depends on "Dark Energy" to power its Universal Acceleration that provides its "simulated gravity". This hypothesis provides no explanation for measured variations in "g".

While the Globe uses well proven gravitation[2] to provide genuine gravity that varies across the earth as explained by rotation and differing polar and equatorial radii and variation with altitude!

The "the flaws inherent in our concepts of the Universe" are simply flaws in your understanding of it. Don't blame Newton, Einstein or anyone else for your failings - go learn a bit about the Globe and more particularly about what alternatives there are! I do not see ANY that explain observations.
Yes, I know you won't consider any of this, but maybe someone else will see the fallacy of the Flat Earth model.

[1] Except right at the poles - they BOTH get 24 hour sunlight on the equinoxes - explain THAT with you Flat Earth sun!
[2] See: "Henry Cavendish proved Newton's Gravitation" http://www.theflatearthsociety.org/forum/index.php?topic=66174.0 (http://www.theflatearthsociety.org/forum/index.php?topic=66174.0)[/list]

First of all, I don't purport to support anything. You only want me to be a flat earther so you can get some satisfaction from relentlessly debating me. As far as I'm concerned you're no better than the flat earthers in regards to ignoring valid points and evidence that might change your world view.

Also, if the universe isn't held together, how can it exist? The heliocentric system relies on a bunch of massive objects whirling around each other, seems it should be utter chaos... Yet it's not. And if you want to accept relativity then you better accept gravity somehow bending time and light and space itself. If you want to accept gravity you better accept it is flawed to the point that relativity had to introduce time dilation and whacky shit like that to hold it together.

What do you really know about our universe? It's a tricky question because I want to know what you first hand, truly know about the universe. Not what Carl Sagan told you, or what you were taught in school. What has anyone? It bothers me to no end to see a bunch of people so god damn sure of themselves when they've only been told these things about our existence, and then they accept it to their core, it's part of their identity now. The difference between a true flat earth believer and a round earth believer is the length of time  they've been entirely too sure about something the scale of which the human mind can hardly fathom.
Title: Re: You wouldn't know how fast you're going
Post by: Unsure101 on March 16, 2016, 12:58:10 PM
It bothers me to no end to see a bunch of people so god damn sure of themselves when they've only been told these things about our existence, and then they accept it to their core, it's part of their identity now.
I've been told that the Grand Canyon exists for as long as I can remember, but I've never seen it. How can I believe that it actually exists as I've never seen it. All pictures of it must be dismissed as fakes and until I see it with my own eyes, it cannot exist. All who say it exists are lying or part of a conspiracy to make me think that it does exist for some reason.
Does this sound a little mad to you?
Title: Re: You wouldn't know how fast you're going
Post by: juner on March 16, 2016, 02:08:40 PM
I've been told that the Grand Canyon exists for as long as I can remember, but I've never seen it. How can I believe that it actually exists as I've never seen it. All pictures of it must be dismissed as fakes and until I see it with my own eyes, it cannot exist. All who say it exists are lying or part of a conspiracy to make me think that it does exist for some reason.
Does this sound a little mad to you?

This is simply a false equivalence.
Title: Re: You wouldn't know how fast you're going
Post by: Rounder on March 17, 2016, 12:55:43 PM
This is simply a false equivalence.

Seems like an apt comparison to me.  The conversations would be very similar to some exchanges here:

The Grand Canyon is real.
     I've seen ditches, I've seen piles of dirt and rocks; dirt and rocks don't pile up into shapes like that.
It isn't a pile of dirt and rocks, it is a channel carved out of solid rock by the Colorado River.
     What?!  Haha, water isn't tougher than ROCK!  Leave a rock in your dishwasher, it won't get any 'canyons' carved in it!
Sure, because it takes millions of years.
     AhHA!  Now I know you've been duped by the GC Conspiracy because the earth isn't that old!
Look at all the photos, online and in printed books and magazines, going way back.
     Photos can be faked in many ways, modern and historic.
But it's a national park!
     Oh, the NPS.  You trust the government, do you?
Lots and lots of people visit it, every year!
     Well, nobody I know has ever been there.
Well, what about the fact that it was surveyed in 1857 by Edward Fitzgerald Beale?
     Since Lewis and Clark didn't see it between 1804-1806, it's probably made up.
Title: Re: You wouldn't know how fast you're going
Post by: Woody on March 17, 2016, 01:53:29 PM
I've been told that the Grand Canyon exists for as long as I can remember, but I've never seen it. How can I believe that it actually exists as I've never seen it. All pictures of it must be dismissed as fakes and until I see it with my own eyes, it cannot exist. All who say it exists are lying or part of a conspiracy to make me think that it does exist for some reason.
Does this sound a little mad to you?

This is simply a false equivalence.

Told the Earth is round.                       Told Grand Canyon exist
Told space travel exist.

Pictures, videos, of Earth                     Dismiss videos and pictures of Grand Canyon
and rockets taking off fake

Can not/never seen curvature               Never seen the Grand Canyon

NASA and likely science in                    Any organization/group saying the Grand Canyon
general part of a conspiracy.                  exist part of a conspiracy.

Conspiracy exist for some ill                  Conspiracy exist for some ill defined reason
defined reason


Seems similar to me. 
Title: Re: You wouldn't know how fast you're going
Post by: TheTruthIsOnHere on March 17, 2016, 03:06:05 PM
You guys are leaving out the fact that most Americans could possibly, you know, go to the Grand Canyon and see it for themselves.

It seems to be a highly selective process to go to space, if you know what I mean. (https://www.google.com/search?q=astronaut+mason+symbols&source=lnms&tbm=isch&sa=X&ved=0ahUKEwil84Wj_8fLAhVLLSYKHawSCjIQ_AUIBygB&biw=1382&bih=730")

Title: Re: You wouldn't know how fast you're going
Post by: BlueMoon on March 17, 2016, 05:30:56 PM
You guys are leaving out the fact that most Americans could possibly, you know, go to the Grand Canyon and see it for themselves.

It seems to be a highly selective process to go to space, if you know what I mean. (http://"<br />https://www.google.com/search?q=astronaut+mason+symbols&source=lnms&tbm=isch&sa=X&ved=0ahUKEwil84Wj_8fLAhVLLSYKHawSCjIQ_AUIBygB&biw=1382&bih=730")
Bad link, but I know what you're getting at.  No, astronauts have nothing to do with freemasons.  It's a highly selective process in that if you don't have years of training, you'll only get in the way.  Fortunately, the astronauts keep us posted on what's going on, so we don't need to feel left out.  And eventually there will be space tourism, though it will probably be prohibitively expensive.  Still, you wouldn't want to spend many thousands of dollars only to get a letter that says "just play along." 
Title: Re: You wouldn't know how fast you're going
Post by: TheTruthIsOnHere on March 17, 2016, 06:15:56 PM
You guys are leaving out the fact that most Americans could possibly, you know, go to the Grand Canyon and see it for themselves.

It seems to be a highly selective process to go to space, if you know what I mean. (http://"<br />https://www.google.com/search?q=astronaut+mason+symbols&source=lnms&tbm=isch&sa=X&ved=0ahUKEwil84Wj_8fLAhVLLSYKHawSCjIQ_AUIBygB&biw=1382&bih=730")
Bad link, but I know what you're getting at.  No, astronauts have nothing to do with freemasons.  It's a highly selective process in that if you don't have years of training, you'll only get in the way.  Fortunately, the astronauts keep us posted on what's going on, so we don't need to feel left out.  And eventually there will be space tourism, though it will probably be prohibitively expensive.  Still, you wouldn't want to spend many thousands of dollars only to get a letter that says "just play along."

The verdict is still out on space tourism. Virgin Galactic looks as though it may never be successful. SpaceX has basically been acquired by NASA, in the form of lucrative contracts... tourism isn't at the top of their list, in the slightest.

Are you debating the fact that many, if not all astronauts have links to freemasonry?
Title: Re: You wouldn't know how fast you're going
Post by: BlueMoon on March 17, 2016, 06:31:12 PM
You guys are leaving out the fact that most Americans could possibly, you know, go to the Grand Canyon and see it for themselves.

It seems to be a highly selective process to go to space, if you know what I mean. (http://"<br />https://www.google.com/search?q=astronaut+mason+symbols&source=lnms&tbm=isch&sa=X&ved=0ahUKEwil84Wj_8fLAhVLLSYKHawSCjIQ_AUIBygB&biw=1382&bih=730")
Bad link, but I know what you're getting at.  No, astronauts have nothing to do with freemasons.  It's a highly selective process in that if you don't have years of training, you'll only get in the way.  Fortunately, the astronauts keep us posted on what's going on, so we don't need to feel left out.  And eventually there will be space tourism, though it will probably be prohibitively expensive.  Still, you wouldn't want to spend many thousands of dollars only to get a letter that says "just play along."

The verdict is still out on space tourism. Virgin Galactic looks as though it may never be successful. SpaceX has basically been acquired by NASA, in the form of lucrative contracts... tourism isn't at the top of their list, in the slightest.

Are you debating the fact that many, if not all astronauts have links to freemasonry?
Yes, I am.  What makes you so certain that they're linked to freemasonry?  Why would they even need to be? 


Also, what about Blue Origin?  They'll be doing suborbital tourism, and they just tested their New Shepard vertical-landing rocket. 
Title: Re: You wouldn't know how fast you're going
Post by: TheTruthIsOnHere on March 17, 2016, 06:50:25 PM
You guys are leaving out the fact that most Americans could possibly, you know, go to the Grand Canyon and see it for themselves.

It seems to be a highly selective process to go to space, if you know what I mean. (http://"<br />https://www.google.com/search?q=astronaut+mason+symbols&source=lnms&tbm=isch&sa=X&ved=0ahUKEwil84Wj_8fLAhVLLSYKHawSCjIQ_AUIBygB&biw=1382&bih=730")
Bad link, but I know what you're getting at.  No, astronauts have nothing to do with freemasons.  It's a highly selective process in that if you don't have years of training, you'll only get in the way.  Fortunately, the astronauts keep us posted on what's going on, so we don't need to feel left out.  And eventually there will be space tourism, though it will probably be prohibitively expensive.  Still, you wouldn't want to spend many thousands of dollars only to get a letter that says "just play along."

The verdict is still out on space tourism. Virgin Galactic looks as though it may never be successful. SpaceX has basically been acquired by NASA, in the form of lucrative contracts... tourism isn't at the top of their list, in the slightest.

Are you debating the fact that many, if not all astronauts have links to freemasonry?
Yes, I am.  What makes you so certain that they're linked to freemasonry?  Why would they even need to be? 


Also, what about Blue Origin?  They'll be doing suborbital tourism, and they just tested their New Shepard vertical-landing rocket.

http://freemasonry.bcy.ca/biography/spacemason/

A short list of free masons that have been in space, as provided by an actual lodge's website.

I could give you more links, and videos showing the rituals and hand shakes and rings and hand signals, but you would probably immediately disregard because of their "conspiratorial" overtones.

Title: Re: You wouldn't know how fast you're going
Post by: BlueMoon on March 17, 2016, 09:15:09 PM
You guys are leaving out the fact that most Americans could possibly, you know, go to the Grand Canyon and see it for themselves.

It seems to be a highly selective process to go to space, if you know what I mean. (http://"<br />https://www.google.com/search?q=astronaut+mason+symbols&source=lnms&tbm=isch&sa=X&ved=0ahUKEwil84Wj_8fLAhVLLSYKHawSCjIQ_AUIBygB&biw=1382&bih=730")
Bad link, but I know what you're getting at.  No, astronauts have nothing to do with freemasons.  It's a highly selective process in that if you don't have years of training, you'll only get in the way.  Fortunately, the astronauts keep us posted on what's going on, so we don't need to feel left out.  And eventually there will be space tourism, though it will probably be prohibitively expensive.  Still, you wouldn't want to spend many thousands of dollars only to get a letter that says "just play along."

The verdict is still out on space tourism. Virgin Galactic looks as though it may never be successful. SpaceX has basically been acquired by NASA, in the form of lucrative contracts... tourism isn't at the top of their list, in the slightest.

Are you debating the fact that many, if not all astronauts have links to freemasonry?
Yes, I am.  What makes you so certain that they're linked to freemasonry?  Why would they even need to be? 


Also, what about Blue Origin?  They'll be doing suborbital tourism, and they just tested their New Shepard vertical-landing rocket.

http://freemasonry.bcy.ca/biography/spacemason/ (http://freemasonry.bcy.ca/biography/spacemason/)

A short list of free masons that have been in space, as provided by an actual lodge's website.

I could give you more links, and videos showing the rituals and hand shakes and rings and hand signals, but you would probably immediately disregard because of their "conspiratorial" overtones.
Look, if they were in such a secret organization, do you really think they would risk using secret rituals and rings and handshakes and the like? 


Furthermore, to be an astronaut you have to be skilled at math, physics, science, and engineering.  The astronauts during the early days of the space program had to be test pilots, and the Apollo astronauts had to be skilled at geology.  Also, they had to be exceptional under pressure and have a good health record.  Do you really think that what few qualified applicants there were would also be freemasons? 


Maybe they were part of their own little fraternity or honor society since joining NASA.  Who knows.  But I'll take their word far and away over a list on any freemason website. 
But I do know that a great many of them were Eagle Scouts, and many others reached another rank in scouting.  Must be the Boy Scouts of America controlling the government  :P
Title: Re: You wouldn't know how fast you're going
Post by: TheTruthIsOnHere on March 17, 2016, 11:32:17 PM
You can keep your eyes closed all you want man the proof is there in black and white. Its not a secret society, because the fact it's not secret. Look at the back of a dollar bill. It's a hide in plain sight type of thing.

And don't forget, I wholesale question the validity of the appolo missions, so the first "astronauts" really didn't need many skills to speak of to be recorded jumping around a movie set.
Title: Re: You wouldn't know how fast you're going
Post by: BlueMoon on March 17, 2016, 11:48:48 PM
You can keep your eyes closed all you want man the proof is there in black and white. Its not a secret society, because the fact it's not secret. Look at the back of a dollar bill. It's a hide in plain sight type of thing.

And don't forget, I wholesale question the validity of the appolo missions, so the first "astronauts" really didn't need many skills to speak of to be recorded jumping around a movie set.


But just because you deny the missions doesn't mean you can ignore the training and accomplishments they had on the ground. 
Like I said, the early astronauts were test pilots.  Best of the best.  How many of them would be freemasons? 


What sort of purpose could hiding an occult symbol on a dollar bill possibly serve??  Is someone going to see the pyramid, and then just go to their nearest Conspiracy Clubhouse and say "I got your invitation, where do I sign up?"


To quote Wikipedia:
Quote
Popular among conspiracy theorists is the claim that the Eye of Providence shown atop an unfinished pyramid on theGreat Seal of the United States indicates the influence of Freemasonry in the founding of the United States. However, common Masonic use of the Eye dates to 14 years after the creation of the Great Seal. Furthermore, among the members of the various design committees for the Great Seal, only Benjamin Franklin was a Mason (and his ideas for the seal were not adopted). Indeed, many Masonic organizations have explicitly denied any connection to the creation of the Seal.
 
Title: Re: You wouldn't know how fast you're going
Post by: TheTruthIsOnHere on March 18, 2016, 12:00:03 AM
Sigh... Do your research or don't. Your choice man. I'm not here to educate or indoctrinate.

The purpose of symbols in general are to imprint ideas and concepts into your head without your knowledge of it happening. A lot of symbols and rites date to prehistory, and in particular, those involving sun worship began to be obvious in Egyptian mythology.

Believe what you will, but ignorance doesn't make you exempt of the influence.
Title: Re: You wouldn't know how fast you're going
Post by: BlueMoon on March 18, 2016, 12:05:55 AM
Sigh... Do your research or don't. Your choice man. I'm not here to educate or indoctrinate.

The purpose of symbols in general are to imprint ideas and concepts into your head without your knowledge of it happening. A lot of symbols and rites date to prehistory, and in particular, those involving sun worship began to be obvious in Egyptian mythology.

Believe what you will, but ignorance doesn't make you exempt of the influence.
Remaining ignorant of your incorrectness will not make you correct. 


I have no desire to discuss this further with you.  Thinking that there is a world organization out to get me will not make a difference; it will only make me paranoid. 
Title: Re: You wouldn't know how fast you're going
Post by: UnionsOfSolarSystemPlanet on March 18, 2016, 12:11:27 AM
Sigh... Do your research or don't. Your choice man. I'm not here to educate or indoctrinate.
YOU of all people tell us to research ourselves? When was the last time you even research of my explanations?

Believe what you will, but ignorance doesn't make you exempt of the influence.
Same goes for you.
Title: Re: You wouldn't know how fast you're going
Post by: TheTruthIsOnHere on March 18, 2016, 12:54:44 AM
But I'm not wrong.

Evidence is there. And trust me I do th research on a lot of the stuff you guys throw at me and if I still have unresolved questions you'll know it.

But the conversation was between mean bluemoon to which it appears he has disrespectfully declined to continue.

The shit was getting too real for him.
Title: Re: You wouldn't know how fast you're going
Post by: rabinoz on March 18, 2016, 01:25:26 AM
But I'm not wrong.
Evidence is there. And trust me I do th research on a lot of the stuff you guys throw at me and if I still have unresolved questions you'll know it.
But the conversation was between mean bluemoon to which it appears he has disrespectfully declined to continue.
The shit was getting too real for him.
Stop pretending that you know what others think and feel! It just possible he has other things to do than to pander to your every whim!
Hope you lurv the formatting you self centred little know it all!
Title: Re: You wouldn't know how fast you're going
Post by: Pete Svarrior on March 18, 2016, 02:05:25 PM
And eventually there will be space tourism, though it will probably be prohibitively expensive.
Oh, how convenient. The thing that makes your equivalency false will eventually not be a problem... but it'll still be horrendously inaccessible.

How very convenient, indeed...

Shall we wait until space tourism actually becomes a thing (which is totally, definitely happening) before you try to compare space to the Grand Canyon?
Title: Re: You wouldn't know how fast you're going
Post by: BlueMoon on March 18, 2016, 04:16:10 PM
And eventually there will be space tourism, though it will probably be prohibitively expensive.
Oh, how convenient. The thing that makes your equivalency false will eventually not be a problem... but it'll still be horrendously inaccessible.

How very convenient, indeed...

Shall we wait until space tourism actually becomes a thing (which is totally, definitely happening) before you try to compare space to the Grand Canyon?
I'm saying once it becomes a regular thing, then will you believe it?  It'll be like a cooler, more expensive form of skydiving, yet it won't be wingsuit jumping, which is also a cooler and (I would imagine) more expensive form of skydiving. 
Title: Re: You wouldn't know how fast you're going
Post by: Rounder on March 18, 2016, 05:30:02 PM
I'm saying once it becomes a regular thing, then will you believe it?

I doubt that will convince anybody.  Tourism to Antarctica is a regular thing now (thousands of 'not scientist' people do it every year) and far more affordable than space tourism is ever likely to be, and yet over and over on this board we still hear variations on the theme of "You can't go to Antarctica...guarded by Navy...forbidden by The Antarctic Treaty..."
Title: Re: You wouldn't know how fast you're going
Post by: TheTruthIsOnHere on March 18, 2016, 05:45:26 PM
I'm saying once it becomes a regular thing, then will you believe it?

I doubt that will convince anybody.  Tourism to Antarctica is a regular thing now (thousands of 'not scientist' people do it every year) and far more affordable than space tourism is ever likely to be, and yet over and over on this board we still hear variations on the theme of "You can't go to Antarctica...guarded by Navy...forbidden by The Antarctic Treaty..."

It's still very cost prohibitive. And it's a closed loop, limited guided tour. You pay $40,000 a person. Fly to one place in Antarctica in a windowless plane, stay in a hut, then leave. Also the main company out of the very very few that do it is a relative of the chief scientist of the NOAA. That is a link to a plausible vested interest.

And as a tourist, you're obviously not going there for a scientific expedition of any sort. You're just trying to find a way to blow some of the money you obviously have too much of.
Title: Re: You wouldn't know how fast you're going
Post by: Rounder on March 18, 2016, 06:40:16 PM
It's still very cost prohibitive. And it's a closed loop, limited guided tour. You pay $40,000 a person. Fly to one place in Antarctica in a windowless plane, stay in a hut, then leave.

According to Antarctic Division of the Australian Government's Department of the Environment (http://www.antarctica.gov.au/about-antarctica/tourism/frequently-asked-questions): "Most visitors reach Antarctica by ship. These voyages last from 10 days to several weeks, and ships range from the basic to the luxurious. Most (except the largest cruise ships, of 500 or more passengers) offer the chance to go ashore." So mostly NOT by plane, windowless or otherwise. 

I'll agree with you on the cost though.  Once you add in travel to and from the embarkation point, one could easily spend $40k or more.


Also the main company out of the very very few that do it is a relative of the chief scientist of the NOAA. That is a link to a plausible vested interest.

Well, "very few" is somewhat misleading.  The International Association of Antarctic Tour Operators lists 45 companies (http://apps.iaato.org/iaato/member/list.xhtml) offering various levels of service to the continent.  Which is NOT an exhaustive list, since the company you are referring to (which is Polar Cruises (http://www.polarcruises.com/) for those who are just joining us) doesn't even appear on the list because they don't actually provide the travel service, they are more of a travel arranger.


And as a tourist, you're obviously not going there for a scientific expedition of any sort. You're just trying to find a way to blow some of the money you obviously have too much of.

True, and on point.  As a tourist, you are not vested in the Conspiracy.  You might see something that you can't reconcile with the RE dogma you've been taught, and even photograph it.  Not knowing the Conspiracy's official explanation, you might blow the whole thing wide open.  This feels to me like an argument against the existence of a Conspiracy, since it would be against their interests to allow anybody not 'in the know' to go to a place where the Conspiracy risks being uncovered.  Therefore, the Conspiracy should have banned tourism when the Antarctic Treaty was signed.  Tourism is not banned.  Therefore either there is no Conspiracy, or the Conspiracy is too weak to influence The Treaty.  If they are too weak to influence The Treaty, how could they at the same time be strong enough to hide the true shape of the Earth from everyone on Earth for hundreds of years?
Title: Re: You wouldn't know how fast you're going
Post by: Woody on March 18, 2016, 07:46:43 PM
I'm saying once it becomes a regular thing, then will you believe it?

I doubt that will convince anybody.  Tourism to Antarctica is a regular thing now (thousands of 'not scientist' people do it every year) and far more affordable than space tourism is ever likely to be, and yet over and over on this board we still hear variations on the theme of "You can't go to Antarctica...guarded by Navy...forbidden by The Antarctic Treaty..."

It's still very cost prohibitive. And it's a closed loop, limited guided tour. You pay $40,000 a person. Fly to one place in Antarctica in a windowless plane, stay in a hut, then leave. Also the main company out of the very very few that do it is a relative of the chief scientist of the NOAA. That is a link to a plausible vested interest.

And as a tourist, you're obviously not going there for a scientific expedition of any sort. You're just trying to find a way to blow some of the money you obviously have too much of.

People go to Antarctica with their own boats.  I ran into a couple in South America that went.  The reason they went is because they ran into another couple who went and told them how beautiful it was.

https://www.google.com/search?q=sailing+antarctica&espv=2&biw=1920&bih=955&source=lnms&tbm=isch&sa=X&ved=0ahUKEwjF_p7u-8rLAhVH22MKHVD7D1MQ_AUIBygC

http://www.bwsailing.com/bw/cruising-news/antarctica-on-our-own/

Here is a guide created by the US government:
http://msi.nga.mil/MSISiteContent/StaticFiles/NAV_PUBS/SD/Pub200/Pub200bk.pdf

I have a boat that I am preparing to go to either North up the west coast or south to Central and South America and on to Antarctica.  I am not rich, but do ok and can afford the trip.  The main reason I can accomplish this is I want to. It seems to me some FE somewhere would want to make the trip.  If it is within my means and ability it is within many people's.
Title: Re: You wouldn't know how fast you're going
Post by: TheTruthIsOnHere on March 18, 2016, 08:52:07 PM
I'm saying once it becomes a regular thing, then will you believe it?

I doubt that will convince anybody.  Tourism to Antarctica is a regular thing now (thousands of 'not scientist' people do it every year) and far more affordable than space tourism is ever likely to be, and yet over and over on this board we still hear variations on the theme of "You can't go to Antarctica...guarded by Navy...forbidden by The Antarctic Treaty..."

It's still very cost prohibitive. And it's a closed loop, limited guided tour. You pay $40,000 a person. Fly to one place in Antarctica in a windowless plane, stay in a hut, then leave. Also the main company out of the very very few that do it is a relative of the chief scientist of the NOAA. That is a link to a plausible vested interest.

And as a tourist, you're obviously not going there for a scientific expedition of any sort. You're just trying to find a way to blow some of the money you obviously have too much of.

People go to Antarctica with their own boats.  I ran into a couple in South America that went.  The reason they went is because they ran into another couple who went and told them how beautiful it was.

https://www.google.com/search?q=sailing+antarctica&espv=2&biw=1920&bih=955&source=lnms&tbm=isch&sa=X&ved=0ahUKEwjF_p7u-8rLAhVH22MKHVD7D1MQ_AUIBygC

http://www.bwsailing.com/bw/cruising-news/antarctica-on-our-own/

Here is a guide created by the US government:
http://msi.nga.mil/MSISiteContent/StaticFiles/NAV_PUBS/SD/Pub200/Pub200bk.pdf

I have a boat that I am preparing to go to either North up the west coast or south to Central and South America and on to Antarctica.  I am not rich, but do ok and can afford the trip.  The main reason I can accomplish this is I want to. It seems to me some FE somewhere would want to make the trip.  If it is within my means and ability it is within many people's.

I hear it's lovely this time of year in Antarctica. /s

I wouldn't waste your time, the main reason I'm sure most people don't go there is because it sucks. Brutal weather, crazy seas, no discernible signs of life. I would go towards Alaska, which despite having similar latitude and sun exposure to regions of Antarctica, is not just a frozen tundra for miles and miles and miles.
Title: Re: You wouldn't know how fast you're going
Post by: nametaken on March 19, 2016, 01:43:20 AM
Edit: I'm a late commer, so this may have been brought up already - I only read OP (for now). Just one thing I got for now:

However, this is not the case.  Consider for a moment that you are riding in a bus.  While it is moving at a constant speed, you get up and move to the other side.  Why don't you get thrown to the back?  The reason is that you retain momentum, and you can only feel acceleration. 

Exactly. However, follow this through to it's logical conclusion, and you realize not only are you right; you are entirely right.

Imagine you jump off that bus, wile it is moving; essentially what we've [allegedly] been doing for years with astronauts. What happens to your organs? Or, is space itself just conveniently moving at the same speed of the Earth, so the 'astronauts' whom have 'already done this' managed to survive, as you were, by 'hitting the ground running'?

Regardless, if the world is spinning and hurtling through space, there should be some sort of 'transition' when exiting the 'vehicle' as you must realize your analogy inevitably must conclude; Yes - you wouldn't know how fast you were going - until it was too late.

Now, obviously, these organizations aren't stupid and would recognize this, and take the measurements into consideration to make a trajectory and exit velocity to 'ease' out of [not knowing how fast they are going]; I'm not saying they didn't do it.
Title: Re: You wouldn't know how fast you're going
Post by: geckothegeek on March 19, 2016, 04:50:01 AM
Flat-earthers often say that it's unbelievable that we would be moving through space so fast. They say that the earth seems solid and stationary, and we would be able to feel if it was whizzing through space or spinning at 1000 mph. 

However, this is not the case.  Consider for a moment that you are riding in a bus.  While it is moving at a constant speed, you get up and move to the other side.  Why don't you get thrown to the back?  The reason is that you retain momentum, and you can only feel acceleration. 

Now, about that "1000 mph" statistic.  The equation for centripetal/centrifugal acceleration is a=v2/r.  The radius r is 6371 km, or 6371000 m.  THe velocity v is about 1000 mph, or 460 m/s.  So our function is 4602/6371000 which gives us...
.033 m/s2
For comparison, acceleration due to gravity at the poles is 9.83 m/s2.  You certainly wouldn't be flung off by that, but it has been measured. 

So why does the atmosphere stay with the earth's surface?  Well, it too has initial momentum.  This confines it to earth's reference frame. 
This initial momentum also explains the Coriolis effect.  Since the surface is spinning faster at the equator, and slower toward the poles, air that moves away from the equator is deflected to the east relative to the surface.  So, if you have an area of low pres***sure, air is drawn toward it, but air from the equator is deflected east, and air from the poles is deflected west.  That causes hurricanes to rotate clockwise in the southern hemisphere and counterclockwise in the northern hemisphere.  The deflection is measurable and consistent, and weather forecasters have to take it into account in their simulations.  It is also visible in the bands and storms of Jupiter. 

So what do you, the Flat Earth Society, have to say about that?  Can you find a better explanation that accounts for the weaker gravity at the equator and the Coriolis effect?  Good luck.

I think this been posted on another subject.Consider the speed of rotation from an "RPM" standpoint. 1 Revolution per 24 hours works out to a very small "Revolutions  Per Minute".
Title: Re: You wouldn't know how fast you're going
Post by: Woody on March 19, 2016, 05:23:14 AM
I'm saying once it becomes a regular thing, then will you believe it?

I doubt that will convince anybody.  Tourism to Antarctica is a regular thing now (thousands of 'not scientist' people do it every year) and far more affordable than space tourism is ever likely to be, and yet over and over on this board we still hear variations on the theme of "You can't go to Antarctica...guarded by Navy...forbidden by The Antarctic Treaty..."

It's still very cost prohibitive. And it's a closed loop, limited guided tour. You pay $40,000 a person. Fly to one place in Antarctica in a windowless plane, stay in a hut, then leave. Also the main company out of the very very few that do it is a relative of the chief scientist of the NOAA. That is a link to a plausible vested interest.

And as a tourist, you're obviously not going there for a scientific expedition of any sort. You're just trying to find a way to blow some of the money you obviously have too much of.

People go to Antarctica with their own boats.  I ran into a couple in South America that went.  The reason they went is because they ran into another couple who went and told them how beautiful it was.

https://www.google.com/search?q=sailing+antarctica&espv=2&biw=1920&bih=955&source=lnms&tbm=isch&sa=X&ved=0ahUKEwjF_p7u-8rLAhVH22MKHVD7D1MQ_AUIBygC

http://www.bwsailing.com/bw/cruising-news/antarctica-on-our-own/

Here is a guide created by the US government:
http://msi.nga.mil/MSISiteContent/StaticFiles/NAV_PUBS/SD/Pub200/Pub200bk.pdf

I have a boat that I am preparing to go to either North up the west coast or south to Central and South America and on to Antarctica.  I am not rich, but do ok and can afford the trip.  The main reason I can accomplish this is I want to. It seems to me some FE somewhere would want to make the trip.  If it is within my means and ability it is within many people's.

I hear it's lovely this time of year in Antarctica. /s

I wouldn't waste your time, the main reason I'm sure most people don't go there is because it sucks. Brutal weather, crazy seas, no discernible signs of life. I would go towards Alaska, which despite having similar latitude and sun exposure to regions of Antarctica, is not just a frozen tundra for miles and miles and miles.

Actually similar to sailing on around the Cape of Good Hope and Cape Horn the reason to go is to challenge yourself.
Title: Re: You wouldn't know how fast you're going
Post by: Rounder on March 19, 2016, 08:25:48 AM
Imagine you jump off that bus, while it is moving....What happens to your organs?

I don't understand; why should anything happen to your organs?

Or, is space itself just conveniently moving at the same speed of the Earth, so the 'astronauts' whom have 'already done this' managed to survive, as you were, by 'hitting the ground running'?

No, it isn't that 'space is moving', but instead that space is empty (OK, it's mostly empty).  When an astronaut leaves the vehicle in space, it isn't like jumping from a moving bus onto stationary earth or into air moving at a different speed (if moving at all).  He's going from a vehicle moving through vacuum, to moving through that vacuum without a vehicle.

Regardless, if the world is spinning and hurtling through space, there should be some sort of 'transition' when exiting the 'vehicle' as you must realize your analogy inevitably must conclude; Yes - you wouldn't know how fast you were going - until it was too late.

You've missed the original poster's point, which is: ON EARTH you cannot detect the velocity of the spot you are standing on, because you detect acceleration, not speed.  If you close your eyes in a moving car, you cannot detect the speed you are travelling.  You can infer it from road noise, wind noise, vibration, etc, but you cannot actually tell how fast you are going.  Once an airliner is up to speed, you don't have any sensation of the forward speed.  You feel the shaking, the up and down, the side to side: in other words, CHANGES in speed, otherwise known as acceleration.  All this subsequent talk of orbiting and leaving spacecraft is all tangential.
Title: Re: You wouldn't know how fast you're going
Post by: TheTruthIsOnHere on March 19, 2016, 02:47:16 PM
Why in videos of the astronauts inside the ISS are they constantly moving around, catching themselves and grabbing stabilization bars? If they aren't experiencing any acceleration then why would that happen?
Title: Re: You wouldn't know how fast you're going
Post by: UnionsOfSolarSystemPlanet on March 19, 2016, 02:56:52 PM
Why in videos of the astronauts inside the ISS are they constantly moving around, catching themselves and grabbing stabilization bars? If they aren't experiencing any acceleration then why would that happen?
They need something to absorb their momentum, once they accelerate to move somewhere, they must decelerate to stop somewhere.
Just look up Newton's first law of motion.
Title: Re: You wouldn't know how fast you're going
Post by: TheTruthIsOnHere on March 19, 2016, 02:58:11 PM
This happens when theyre seemingly standing still, not just demos of them moving around the station.
Title: Re: You wouldn't know how fast you're going
Post by: UnionsOfSolarSystemPlanet on March 19, 2016, 03:13:00 PM
This happens when theyre seemingly standing still, not just demos of them moving around the station.
Because they aren't standing still, you probably mention how they slowly rotates.
Do a test yourself, jump while turning your body quickly, can you stop the rotation while on air?
When your feet touches the ground, it will absorb your momentum, but in the ISS, they need to hold on something connected to the station to do that.
Title: Re: You wouldn't know how fast you're going
Post by: BlueMoon on March 19, 2016, 03:19:25 PM
Why in videos of the astronauts inside the ISS are they constantly moving around, catching themselves and grabbing stabilization bars? If they aren't experiencing any acceleration then why would that happen?
When in the microgravity of the space station, any motion that they make causes them to move or rotate.  There is no ground for them to plant their feet on.  They also have air currents to contend with. 
Title: Re: You wouldn't know how fast you're going
Post by: andruszkow on March 19, 2016, 03:27:19 PM
Also, how would you ever be completely still on ISS? Turning your head of moving your arm will cause you to move. Even the blood being pumped around your body will cause you to move eventually
Title: Re: You wouldn't know how fast you're going
Post by: nametaken on March 19, 2016, 05:07:37 PM
I don't understand; why should anything happen to your organs?

You cleared this up for me; but here's where I was coming from, and what I thought would happen prior: Same thing when you slam on the brakes in a vehicle, or deploy a parachute I suppose, just on a bigger scale of being suddenly detached from the earth's movement. End: It was relational I see now; there's nothing to move in relation to in space really (until inevitably of course, there is, such as debris or larger), so no violent transition.

However, if you're in a car there are a lot of inferences you can make about the car's movement in relation to the known Globe Earth momentum; even with a majority of the senses relatively impaired (close your eyes, put in earplugs, etc). This is because your body (and specifically) organs (again) can detect changes in movement. Try taking a cab when you are looking for a latrine, for example; experiencing turbulence on an aircraft or vertigo at night sky may apply to some extent as well. However, this may/may not apply to constant movement which is all said organs have ever known since before they were even developed (save for in my 'tangent' about leaving it); such as the movement of the Globe Earth. I honestly don't think it can even be used to prove/disprove the theory of this topic (except again, by observations/measurements leaving it). Hope that helps; actually would be an interesting (if expensive) experiment to run in addition to any other task in a manned mission, actually; probably not worth going to space alone just to find out though.

Or, is space itself just conveniently moving at the same speed of the Earth, so the 'astronauts' whom have 'already done this' managed to survive, as you were, by 'hitting the ground running'?

No, it isn't that 'space is moving', but instead that space is empty (OK, it's mostly empty).  When an astronaut leaves the vehicle in space, it isn't like jumping from a moving bus onto stationary earth or into air moving at a different speed (if moving at all).  He's going from a vehicle moving through vacuum, to moving through that vacuum without a vehicle.

Okay, I see how these globe-earth-to-space-transition physics work better now, thank you, this is the counterpoint I couldn't visualize (thus my point of my last post). I by no means was implying that vacuum had momentum, though; that was just to illustrate what I couldn't visualize.

Regardless, if the world is spinning and hurtling through space, there should be some sort of 'transition' when exiting the 'vehicle' as you must realize your analogy inevitably must conclude; Yes - you wouldn't know how fast you were going - until [you stopped].

You've missed the original poster's point, which is: ON EARTH you cannot detect the velocity of the spot you are standing on, because you detect acceleration, not speed [...] All this subsequent talk of orbiting and leaving spacecraft is all tangential.

My previous quoted response. I didn't miss anything other than transition physics; Edit de/acceleration opposed to speed, and over-interpreted organ impact. End Edit Oh yes, it was [somewhat of] a tangent, maybe in bad taste, I concur, but would have been worse to make a new thread for it entirely than to merely bring it up here (to be sure, just to be thwarted with a single post such as this - thank you for responding btw).

« Last Edit: Today at 05:14:16 PM by nametaken »
Title: Re: You wouldn't know how fast you're going
Post by: BCGreenwood on April 07, 2016, 10:55:39 AM
According to flat earth theory I am travelling faster than the speed of light.

(Flat Earth wiki)
Why doesn't gravity pull the earth into a spherical shape?
The earth isn't pulled into a sphere because the force known as gravity doesn't exist or at least exists in a greatly diminished form than is commonly taught. The earth is constantly accelerating up at a rate of 32 feet per second squared (or 9.8 meters per second squared). This constant acceleration causes what you think of as gravity.
(Flat Earth wiki)

After a single year the Earth would have been moving faster than the speed of light.

Breaking the laws of physics aside, my question is :) why don't you feel the effects of moving so fast?
Title: Re: You wouldn't know how fast you're going
Post by: juner on April 07, 2016, 04:54:26 PM
According to flat earth theory I am travelling faster than the speed of light.

(Flat Earth wiki)
Why doesn't gravity pull the earth into a spherical shape?
The earth isn't pulled into a sphere because the force known as gravity doesn't exist or at least exists in a greatly diminished form than is commonly taught. The earth is constantly accelerating up at a rate of 32 feet per second squared (or 9.8 meters per second squared). This constant acceleration causes what you think of as gravity.
(Flat Earth wiki)

After a single year the Earth would have been moving faster than the speed of light.

Breaking the laws of physics aside, my question is :) why don't you feel the effects of moving so fast?

No, the Earth wouldn't be moving faster than the speed of light. Not after a year, not ever.

You do feel the effects. Try jumping or stepping off a chair.
Title: Re: You wouldn't know how fast you're going
Post by: BCGreenwood on April 07, 2016, 05:24:33 PM
So you disagree with the Flat Earth wiki?
That seemed pretty clear that the Earth was constantly accelerating, which is the reason why I will hit the floor after stepping off a chair.

However, if the Earth was constantly accelerating like this after just a year it would be moving faster than the speed of light. My question is why don't I feel like I'm moving a few thousand times the speed of light when I'm simply sitting in the chair?
Title: Re: You wouldn't know how fast you're going
Post by: TheTruthIsOnHere on April 07, 2016, 05:25:43 PM
So you disagree with the Flat Earth wiki?
That seemed pretty clear that the Earth was constantly accelerating, which is the reason why I will hit the floor after stepping off a chair.

However, if the Earth was constantly accelerating like this after just a year it would be moving faster than the speed of light. My question is why don't I feel like I'm moving a few thousand times the speed of light when I'm simply sitting in the chair?

Because you wouldn't know how fast you're going ;) lol
Title: Re: You wouldn't know how fast you're going
Post by: BCGreenwood on April 07, 2016, 05:53:36 PM
Do you think you're travelling over 5000 times the speed of light? (Over 1.5 Billion km a second)

I feel like I'd know if I was moving that fast.
Title: You wouldn't know how fast you're going
Post by: juner on April 07, 2016, 07:25:39 PM
So you disagree with the Flat Earth wiki?
That seemed pretty clear that the Earth was constantly accelerating, which is the reason why I will hit the floor after stepping off a chair.

However, if the Earth was constantly accelerating like this after just a year it would be moving faster than the speed of light. My question is why don't I feel like I'm moving a few thousand times the speed of light when I'm simply sitting in the chair?

No, I don't disagree with the wiki. You aren't moving faster than the speed of light. You can accelerate forever at a constant rate and never reach c. You will asymptotically approach c, but never reach it. I would suggest taking a look at special relativity for more detail. Even the wiki explains why you won't reach c.
Title: Re: You wouldn't know how fast you're going
Post by: BCGreenwood on April 07, 2016, 07:42:40 PM
I had only read the part on gravity that I quoted, I never even imagined that Flat Earthers would believe in special relativity and that E=Mc2.

So you're saying you believe the explanation that our mass is nearing infinity?

(Our mass would have to be nearing infinity if we were travelling at the speed that we were experiencing length contraction.)
Title: Re: You wouldn't know how fast you're going
Post by: Roundy on April 10, 2016, 12:47:16 AM
I had only read the part on gravity that I quoted, I never even imagined that Flat Earthers would believe in special relativity and that E=Mc2.

So you're saying you believe the explanation that our mass is nearing infinity?

That's just silly.  It is a ridiculous impossibility for our mass to be "nearing infinity".
Title: Re: You wouldn't know how fast you're going
Post by: Unsure101 on April 10, 2016, 05:04:23 AM
I had only read the part on gravity that I quoted, I never even imagined that Flat Earthers would believe in special relativity and that E=Mc2.

So you're saying you believe the explanation that our mass is nearing infinity?

That's just silly.  It is a ridiculous impossibility for our mass to be "nearing infinity".
Then how do you explain that we've been accelerating for so long, but our velocity is less that the speed of light?
Title: Re: You wouldn't know how fast you're going
Post by: Roundy on April 10, 2016, 06:00:37 AM
I had only read the part on gravity that I quoted, I never even imagined that Flat Earthers would believe in special relativity and that E=Mc2.

So you're saying you believe the explanation that our mass is nearing infinity?

That's just silly.  It is a ridiculous impossibility for our mass to be "nearing infinity".
Then how do you explain that we've been accelerating for so long, but our velocity is less that the speed of light?

Because you can constantly accelerate without reaching the speed of light.  It is impossible to reach the speed of light.  I thought you had already grasped this.
Title: Re: You wouldn't know how fast you're going
Post by: BCGreenwood on April 10, 2016, 06:34:38 AM
I had only read the part on gravity that I quoted, I never even imagined that Flat Earthers would believe in special relativity and that E=Mc2.

So you're saying you believe the explanation that our mass is nearing infinity?

That's just silly.  It is a ridiculous impossibility for our mass to be "nearing infinity".
Then how do you explain that we've been accelerating for so long, but our velocity is less that the speed of light?

Because you can constantly accelerate without reaching the speed of light.  It is impossible to reach the speed of light.  I thought you had already grasped this.

The part you haven't grasped is that if we were moving so fast to experience length contraction (and therefore never reach the speed of light) our mass would have increased close to infinity. That's all part of special relativity.

I'm glad you think the flat earth wiki is silly and ridiculous, I feel the same.
Title: Re: You wouldn't know how fast you're going
Post by: Roundy on April 10, 2016, 06:40:17 AM
I had only read the part on gravity that I quoted, I never even imagined that Flat Earthers would believe in special relativity and that E=Mc2.

So you're saying you believe the explanation that our mass is nearing infinity?

That's just silly.  It is a ridiculous impossibility for our mass to be "nearing infinity".
Then how do you explain that we've been accelerating for so long, but our velocity is less that the speed of light?

Because you can constantly accelerate without reaching the speed of light.  It is impossible to reach the speed of light.  I thought you had already grasped this.

The part you haven't grasped is that if we were moving so fast to experience length contraction (and therefore never reach the speed of light) our mass would have increased close to infinity. That's all part of special relativity.


Why do you keep using the phrase "close to infinity" as if it has any meaning at all?  At what point exactly does one reach "close to infinity"?
Title: Re: You wouldn't know how fast you're going
Post by: BCGreenwood on April 10, 2016, 06:54:36 AM
I had only read the part on gravity that I quoted, I never even imagined that Flat Earthers would believe in special relativity and that E=Mc2.

So you're saying you believe the explanation that our mass is nearing infinity?

That's just silly.  It is a ridiculous impossibility for our mass to be "nearing infinity".
Then how do you explain that we've been accelerating for so long, but our velocity is less that the speed of light?

Because you can constantly accelerate without reaching the speed of light.  It is impossible to reach the speed of light.  I thought you had already grasped this.

The part you haven't grasped is that if we were moving so fast to experience length contraction (and therefore never reach the speed of light) our mass would have increased close to infinity. That's all part of special relativity.


Why do you keep using the phrase "close to infinity" as if it has any meaning at all?  At what point exactly does one reach "close to infinity"?

Again this is part of special relativity, the thing you're using to explain what is happening.
Briefly:
In order to reach the speed of light your mass would be infinite.
In order to experience length contraction (the reason you give for not reaching the speed of light) you have to be travelling at 99.9999999% of the speed of light.
Therefore your mass is getting close to infinite.

If it's just the concept of infinity you don't like, I can explain things in a simpler way for you.
You're trying to argue that there is more mass in the palm of your hand than there is in a black hole.

Sounds crazy, I know.
Title: Re: You wouldn't know how fast you're going
Post by: BlueMoon on April 10, 2016, 08:07:58 AM
I had only read the part on gravity that I quoted, I never even imagined that Flat Earthers would believe in special relativity and that E=Mc2.

So you're saying you believe the explanation that our mass is nearing infinity?

That's just silly.  It is a ridiculous impossibility for our mass to be "nearing infinity".
Then how do you explain that we've been accelerating for so long, but our velocity is less that the speed of light?

Because you can constantly accelerate without reaching the speed of light.  It is impossible to reach the speed of light.  I thought you had already grasped this.

The part you haven't grasped is that if we were moving so fast to experience length contraction (and therefore never reach the speed of light) our mass would have increased close to infinity. That's all part of special relativity.


Why do you keep using the phrase "close to infinity" as if it has any meaning at all?  At what point exactly does one reach "close to infinity"?
You're right about accelerating without reaching the speed of light.  In that one regard, and from my limited knowledge of relativity, it checks out. 
Title: Re: You wouldn't know how fast you're going
Post by: Round fact on April 10, 2016, 11:16:15 AM
In general the standard pressure lapse rate is about an inch of Hg per thousand feet of altitude gained. And for temperature, about 4.5 degrees F.

This assumes a sea level pressure of 29.92 and Temp of 59 degrees which being a world wide average, means your location will more often that not be different than the Standard.

Our airport here in Columbus, Indiana is 658 feet above sea level. So our temperature should be about 57 degrees and the pressure 29.26. But weather gets involved. I have seen in the winter sub-zero temps and low pressure fronts combine to give our airport pressure and density altitudes of 4,000 feet BELOW Sea Level.  On those days a little Cessna 150 will jump off the runway like it sat on ten hornet's nests. I have seen the reverse on a hot clear summer's day where the aircraft lumbers down the runway as though it will never take off.

Read articles about the Leadville  CO airport. Very enlightening about temperature and pressure.

Though the excepted beginning of Space is 62 miles altitude its influence extends well above that as the atmosphere has no defined edge, and in truth exerts a small amount of drag on the ISS and other low Earth orbit sats. This is the reason Sky Lab and the Russian Station finally fell out of orbit and burned up (mostly) as it entered the atmosphere.
Title: Re: You wouldn't know how fast you're going
Post by: Roundy on April 10, 2016, 04:43:57 PM
Quote
Why do you keep using the phrase "close to infinity" as if it has any meaning at all?  At what point exactly does one reach "close to infinity"?

Again this is part of special relativity, the thing you're using to explain what is happening.
Briefly:
In order to reach the speed of light your mass would be infinite.
In order to experience length contraction (the reason you give for not reaching the speed of light) you have to be travelling at 99.9999999% of the speed of light.
Therefore your mass is getting close to infinite.

If it's just the concept of infinity you don't like, I can explain things in a simpler way for you.
You're trying to argue that there is more mass in the palm of your hand than there is in a black hole.

Sounds crazy, I know.

I understand the concept of infinity well enough to know that it is impossible to ever reach it, therefore it is impossible to ever get close to it.  Sounds crazy, I know  ::), but if you really put some thought into that you might grasp it.  It is an extremely simple concept.
Title: Re: You wouldn't know how fast you're going
Post by: Round fact on April 10, 2016, 05:01:38 PM
Quote
Why do you keep using the phrase "close to infinity" as if it has any meaning at all?  At what point exactly does one reach "close to infinity"?

Again this is part of special relativity, the thing you're using to explain what is happening.
Briefly:
In order to reach the speed of light your mass would be infinite.
In order to experience length contraction (the reason you give for not reaching the speed of light) you have to be travelling at 99.9999999% of the speed of light.
Therefore your mass is getting close to infinite.

If it's just the concept of infinity you don't like, I can explain things in a simpler way for you.
You're trying to argue that there is more mass in the palm of your hand than there is in a black hole.

Sounds crazy, I know.

I understand the concept of infinity well enough to know that it is impossible to ever reach it, therefore it is impossible to ever get close to it.  Sounds crazy, I know  ::), but if you really put some thought into that you might grasp it.  It is an extremely simple concept.

Accelerating at 1g for 100 years will get you to  0.9998192668726723 c. In my book that is close to infinite
Title: Re: You wouldn't know how fast you're going
Post by: BCGreenwood on April 10, 2016, 05:04:21 PM
Quote
Why do you keep using the phrase "close to infinity" as if it has any meaning at all?  At what point exactly does one reach "close to infinity"?

Again this is part of special relativity, the thing you're using to explain what is happening.
Briefly:
In order to reach the speed of light your mass would be infinite.
In order to experience length contraction (the reason you give for not reaching the speed of light) you have to be travelling at 99.9999999% of the speed of light.
Therefore your mass is getting close to infinite.

If it's just the concept of infinity you don't like, I can explain things in a simpler way for you.
You're trying to argue that there is more mass in the palm of your hand than there is in a black hole.

Sounds crazy, I know.

I understand the concept of infinity well enough to know that it is impossible to ever reach it, therefore it is impossible to ever get close to it.  Sounds crazy, I know  ::), but if you really put some thought into that you might grasp it.  It is an extremely simple concept.


So you don't understand special relativity, that's ok and honestly it's what I expected.
That's why I dumbed it down for you but for some reason you don't want to talk about it, I wonder what that reason might be.

Maybe you just like the idea of the palm of your hand having more mass than a black hole. I'm sure you don't have any proof of that but I doubt that bothers you, you strike me as someone that doesn't need proof for what they believe.

So are you going to back up what you believe or just dodge again?
Bear in mind I'm predicting dodge.
Title: Re: You wouldn't know how fast you're going
Post by: Roundy on April 10, 2016, 05:51:38 PM
Quote
Why do you keep using the phrase "close to infinity" as if it has any meaning at all?  At what point exactly does one reach "close to infinity"?

Again this is part of special relativity, the thing you're using to explain what is happening.
Briefly:
In order to reach the speed of light your mass would be infinite.
In order to experience length contraction (the reason you give for not reaching the speed of light) you have to be travelling at 99.9999999% of the speed of light.
Therefore your mass is getting close to infinite.

If it's just the concept of infinity you don't like, I can explain things in a simpler way for you.
You're trying to argue that there is more mass in the palm of your hand than there is in a black hole.

Sounds crazy, I know.

I understand the concept of infinity well enough to know that it is impossible to ever reach it, therefore it is impossible to ever get close to it.  Sounds crazy, I know  ::), but if you really put some thought into that you might grasp it.  It is an extremely simple concept.

Accelerating at 1g for 100 years will get you to  0.9998192668726723 c. In my book that is close to infinite

No, it's close to c (which is not infinity).  And it's not a valid argument regarding the mass of the objects traveling at that speed because numbers don't work that way.  Whatever mass an object would be at that speed, it could be five times as much, it could be a thousand times as much.  And that goes for any speed.  The phrase "close to infinite" has no intrinsic meaning; it is an absurdity.

So you don't understand special relativity, that's ok and honestly it's what I expected.

Oh, the irony!
Title: Re: You wouldn't know how fast you're going
Post by: BCGreenwood on April 10, 2016, 05:59:47 PM
Quote
Why do you keep using the phrase "close to infinity" as if it has any meaning at all?  At what point exactly does one reach "close to infinity"?

Again this is part of special relativity, the thing you're using to explain what is happening.
Briefly:
In order to reach the speed of light your mass would be infinite.
In order to experience length contraction (the reason you give for not reaching the speed of light) you have to be travelling at 99.9999999% of the speed of light.
Therefore your mass is getting close to infinite.

If it's just the concept of infinity you don't like, I can explain things in a simpler way for you.
You're trying to argue that there is more mass in the palm of your hand than there is in a black hole.

Sounds crazy, I know.

I understand the concept of infinity well enough to know that it is impossible to ever reach it, therefore it is impossible to ever get close to it.  Sounds crazy, I know  ::), but if you really put some thought into that you might grasp it.  It is an extremely simple concept.

Accelerating at 1g for 100 years will get you to  0.9998192668726723 c. In my book that is close to infinite

No, it's close to c (which is not infinity).  And it's not a valid argument regarding the mass of the objects traveling at that speed because numbers don't work that way.  Whatever mass an object would be at that speed, it could be five times as much, it could be a thousand times as much.  And that goes for any speed.  The phrase "close to infinite" has no intrinsic meaning; it is an absurdity.

So you don't understand special relativity, that's ok and honestly it's what I expected.

Oh, the irony!

Great so you admit that the speed you think the Earth is travelling at is close to the speed of light.

Spoilers
In special relativity as an object's velocity (speed) approaches the speed of light the mass of that object increases. If it reached the speed of light it's mass would be infinite.

Congratulations, you've admitted the Earth's mass is approaching infinite.
Although not much of a congratulations because you're still wrong.
Title: Re: You wouldn't know how fast you're going
Post by: Rounder on April 10, 2016, 06:37:14 PM
I understand the concept of infinity well enough to know that it is impossible to ever reach it....It is an extremely simple concept.

Evidence that you DON'T, in fact, understand it.  Infinity is an incredibly complex concept.  Take just for starters the fact that one infinite set (the set of all integers, for example) can be larger than another infinite set (the set of all ODD integers).  Both sets are infinite, but one is twice as big as the other.  Or is it twice as big?  I don't know, neither do you, and neither do PhD mathemeticians!
Title: Re: You wouldn't know how fast you're going
Post by: juner on April 12, 2016, 02:44:28 AM
I had only read the part on gravity that I quoted, I never even imagined that Flat Earthers would believe in special relativity and that E=Mc2.

So you're saying you believe the explanation that our mass is nearing infinity?

(Our mass would have to be nearing infinity if we were travelling at the speed that we were experiencing length contraction.)

That isn't how that works. You know it depends on what you mean by "mass," right? Inertial mass increases, which requires more energy to continue acceleration. You are over simplifying the relationship between mass and energy in this instance. I would suggest checking out https://en.wikipedia.org/wiki/Mass_in_special_relativity
Title: Re: You wouldn't know how fast you're going
Post by: BCGreenwood on April 12, 2016, 04:55:45 AM
I had only read the part on gravity that I quoted, I never even imagined that Flat Earthers would believe in special relativity and that E=Mc2.

So you're saying you believe the explanation that our mass is nearing infinity?

(Our mass would have to be nearing infinity if we were travelling at the speed that we were experiencing length contraction.)

That isn't how that works. You know it depends on what you mean by "mass," right? Inertial mass increases, which requires more energy to continue acceleration. You are over simplifying the relationship between mass and energy in this instance. I would suggest checking out https://en.wikipedia.org/wiki/Mass_in_special_relativity

If you're disagreeing with what I'm saying I would suggest YOU actually read that same entry on special relativity.

If you still think I'm wrong I'm more than happy to talk about where our understanding of special relativity differs but obviously you'll have to be slightly more specific than "that isn't how that works".

Just so we're clear, you're taking the stance that the Earth is travelling at 99.9% of the speed of light right now. And right now we're experiencing length contraction because of the speed we're currently travelling.

I know that's pretty much given from your argument I just wanted that as a nice little base to work off.
Title: Re: You wouldn't know how fast you're going
Post by: rabinoz on April 12, 2016, 06:54:48 AM
Quote
Why do you keep using the phrase "close to infinity" as if it has any meaning at all?  At what point exactly does one reach "close to infinity"?

Again this is part of special relativity, the thing you're using to explain what is happening.
Briefly:
In order to reach the speed of light your mass would be infinite.
In order to experience length contraction (the reason you give for not reaching the speed of light) you have to be travelling at 99.9999999% of the speed of light.
Therefore your mass is getting close to infinite.

If it's just the concept of infinity you don't like, I can explain things in a simpler way for you.
You're trying to argue that there is more mass in the palm of your hand than there is in a black hole.

Sounds crazy, I know.

I understand the concept of infinity well enough to know that it is impossible to ever reach it, therefore it is impossible to ever get close to it.  Sounds crazy, I know  ::), but if you really put some thought into that you might grasp it.  It is an extremely simple concept.

Accelerating at 1g for 100 years will get you to  0.9998192668726723 c. In my book that is close to infinite
I know I'm a bit late telling you (btw did your value come from "Relativistic Star Ship Calculator"), but the 100 years you quote is for the time elapsed in the inertial FOR, on the accelerating earth the time elapsed is only about 8.986 years! (Time dilation and all that stuff).

Just try to find the Earth's velocity and time elapsed in the inertial FOR for say 6020 years earth time!
Title: Re: You wouldn't know how fast you're going
Post by: Round fact on April 12, 2016, 11:03:59 AM
Quote
Why do you keep using the phrase "close to infinity" as if it has any meaning at all?  At what point exactly does one reach "close to infinity"?

Again this is part of special relativity, the thing you're using to explain what is happening.
Briefly:
In order to reach the speed of light your mass would be infinite.
In order to experience length contraction (the reason you give for not reaching the speed of light) you have to be travelling at 99.9999999% of the speed of light.
Therefore your mass is getting close to infinite.

If it's just the concept of infinity you don't like, I can explain things in a simpler way for you.
You're trying to argue that there is more mass in the palm of your hand than there is in a black hole.

Sounds crazy, I know.

I understand the concept of infinity well enough to know that it is impossible to ever reach it, therefore it is impossible to ever get close to it.  Sounds crazy, I know  ::), but if you really put some thought into that you might grasp it.  It is an extremely simple concept.

Accelerating at 1g for 100 years will get you to  0.9998192668726723 c. In my book that is close to infinite
I know I'm a bit late telling you (btw did your value come from "Relativistic Star Ship Calculator"), but the 100 years you quote is for the time elapsed in the inertial FOR, on the accelerating earth the time elapsed is only about 8.986 years! (Time dilation and all that stuff).

Just try to find the Earth's velocity and time elapsed in the inertial FOR for say 6020 years earth time!

I know what you are saying and why and I agree with you. My point was that under UA we have had to reach 99.9% c and our mass would be .01 from infinite. I certainly didn't mean to muddy the water here.
Title: Re: You wouldn't know how fast you're going
Post by: juner on April 12, 2016, 03:41:14 PM
If you're disagreeing with what I'm saying I would suggest YOU actually read that same entry on special relativity.
So, you didn't read it, then. Gotcha.


Quote
If you still think I'm wrong I'm more than happy to talk about where our understanding of special relativity differs but obviously you'll have to be slightly more specific than "that isn't how that works".
You are talking about mass in absolute terms. Saying something like "nearing infinity" in this discussion is irrelevant. You obviously don't have a good enough understanding of SR to try and debunk UA with it. The model certainly isn't perfect, and there are other ways to scrutinize or criticize it. I suggest you look into those.


Quote
Just so we're clear, you're taking the stance that the Earth is travelling at 99.9% of the speed of light right now. And right now we're experiencing length contraction because of the speed we're currently travelling.
Again, this is a meaningless statement.
Title: Re: You wouldn't know how fast you're going
Post by: Round fact on April 12, 2016, 03:54:55 PM

Quote
You are talking about mass in absolute terms. Saying something like "nearing infinity" in this discussion is irrelevant. You obviously don't have a good enough understanding of SR to try and debunk UA with it. The model certainly isn't perfect, and there are other ways to scrutinize or criticize it. I suggest you look into those.

In other words you cannot show how it is irrelevant to UA, so you need to move on.

Math is the straw that breaks the back of FE in all cases. You cannot just cherry pick what you think fits and declare the rest of math irrelevant and the issue settled. UA, perspective, spotlight sun, not spotlight sun, none of stand up to the math. And when the issue of math is FORCED into the thread subject, you either cherry pick the .000001% that fits or ignore it altogether. 
Title: Re: You wouldn't know how fast you're going
Post by: juner on April 12, 2016, 04:11:42 PM
In other words you cannot show how it is irrelevant to UA, so you need to move on.
If you had the capacity to pay attention, you would see that was already done. It is you that needs to move on since you either don't understand it, or refuse to try. If you have a question, I can help you out, but you would rather make nonsensical claims than have an actual discussion.

Quote
Math is the straw that breaks the back of FE in all cases.
This is objectively false.

Quote
You cannot just cherry pick what you think fits and declare the rest of math irrelevant and the issue settled.
That isn't happening. SR explains the topic at hand quite nicely. I think it is you who doesn't understand the maths.

Quote
UA, perspective, spotlight sun, not spotlight sun, none of stand up to the math. And when the issue of math is FORCED into the thread subject, you either cherry pick the .000001% that fits or ignore it altogether.
Also objectively false.
Title: Re: You wouldn't know how fast you're going
Post by: BCGreenwood on April 12, 2016, 04:17:46 PM
If you're disagreeing with what I'm saying I would suggest YOU actually read that same entry on special relativity.
So, you didn't read it, then. Gotcha.


Quote
If you still think I'm wrong I'm more than happy to talk about where our understanding of special relativity differs but obviously you'll have to be slightly more specific than "that isn't how that works".
You are talking about mass in absolute terms. Saying something like "nearing infinity" in this discussion is irrelevant. You obviously don't have a good enough understanding of SR to try and debunk UA with it. The model certainly isn't perfect, and there are other ways to scrutinize or criticize it. I suggest you look into those.


Quote
Just so we're clear, you're taking the stance that the Earth is travelling at 99.9% of the speed of light right now. And right now we're experiencing length contraction because of the speed we're currently travelling.
Again, this is a meaningless statement.


So again you can't actually say where I'm wrong, I'm sensing a pattern here where you just repeatedly dodge the question.

Just because you don't understand the statement doesn't make it meaningless. Well maybe meaningless to you because you don't understand it.

I can try and help you understand it if you'd like. It is a pretty simple statement already but if you tell me which part you're struggling with I can explain it to you.
Title: Re: You wouldn't know how fast you're going
Post by: juner on April 12, 2016, 04:22:50 PM
If you're disagreeing with what I'm saying I would suggest YOU actually read that same entry on special relativity.
So, you didn't read it, then. Gotcha.


Quote
If you still think I'm wrong I'm more than happy to talk about where our understanding of special relativity differs but obviously you'll have to be slightly more specific than "that isn't how that works".
You are talking about mass in absolute terms. Saying something like "nearing infinity" in this discussion is irrelevant. You obviously don't have a good enough understanding of SR to try and debunk UA with it. The model certainly isn't perfect, and there are other ways to scrutinize or criticize it. I suggest you look into those.


Quote
Just so we're clear, you're taking the stance that the Earth is travelling at 99.9% of the speed of light right now. And right now we're experiencing length contraction because of the speed we're currently travelling.
Again, this is a meaningless statement.


So again you can't actually say where I'm wrong, I'm sensing a pattern here where you just repeatedly dodge the question.

Just because you don't understand the statement doesn't make it meaningless. Well maybe meaningless to you because you don't understand it.

I can try and help you understand it if you'd like. It is a pretty simple statement already but if you tell me which part you're struggling with I can explain it to you.

I've explained it to you. I don't know how much you know about SR, so I'm not sure where you want me to start helping you. It seems you'd rather just continue to make baseless claims, presumably because it makes you feel superior. It is clear you have little grasp on the topic, so instead of trying to discuss, you deflect and act like nothing is being said. It's bordering on being intellectually dishonest. Feel free to come back when you want to have an actual discussion.
Title: Re: You wouldn't know how fast you're going
Post by: BCGreenwood on April 12, 2016, 04:59:50 PM
If you're disagreeing with what I'm saying I would suggest YOU actually read that same entry on special relativity.
So, you didn't read it, then. Gotcha.


Quote
If you still think I'm wrong I'm more than happy to talk about where our understanding of special relativity differs but obviously you'll have to be slightly more specific than "that isn't how that works".
You are talking about mass in absolute terms. Saying something like "nearing infinity" in this discussion is irrelevant. You obviously don't have a good enough understanding of SR to try and debunk UA with it. The model certainly isn't perfect, and there are other ways to scrutinize or criticize it. I suggest you look into those.


Quote
Just so we're clear, you're taking the stance that the Earth is travelling at 99.9% of the speed of light right now. And right now we're experiencing length contraction because of the speed we're currently travelling.
Again, this is a meaningless statement.


So again you can't actually say where I'm wrong, I'm sensing a pattern here where you just repeatedly dodge the question.

Just because you don't understand the statement doesn't make it meaningless. Well maybe meaningless to you because you don't understand it.

I can try and help you understand it if you'd like. It is a pretty simple statement already but if you tell me which part you're struggling with I can explain it to you.

I've explained it to you. I don't know how much you know about SR, so I'm not sure where you want me to start helping you. It seems you'd rather just continue to make baseless claims, presumably because it makes you feel superior. It is clear you have little grasp on the topic, so instead of trying to discuss, you deflect and act like nothing is being said. It's bordering on being intellectually dishonest. Feel free to come back when you want to have an actual discussion.

And once again you just dodge the question, surprise surprise.

Okay then, I guess if I try to explain it in little baby steps for you, you might stand a chance.

Do you understand why flat earthers think the Earth can continuously accelerate but never reach the speed of light?
Title: Re: You wouldn't know how fast you're going
Post by: juner on April 12, 2016, 05:06:59 PM
If you're disagreeing with what I'm saying I would suggest YOU actually read that same entry on special relativity.
So, you didn't read it, then. Gotcha.


Quote
If you still think I'm wrong I'm more than happy to talk about where our understanding of special relativity differs but obviously you'll have to be slightly more specific than "that isn't how that works".
You are talking about mass in absolute terms. Saying something like "nearing infinity" in this discussion is irrelevant. You obviously don't have a good enough understanding of SR to try and debunk UA with it. The model certainly isn't perfect, and there are other ways to scrutinize or criticize it. I suggest you look into those.


Quote
Just so we're clear, you're taking the stance that the Earth is travelling at 99.9% of the speed of light right now. And right now we're experiencing length contraction because of the speed we're currently travelling.
Again, this is a meaningless statement.


So again you can't actually say where I'm wrong, I'm sensing a pattern here where you just repeatedly dodge the question.

Just because you don't understand the statement doesn't make it meaningless. Well maybe meaningless to you because you don't understand it.

I can try and help you understand it if you'd like. It is a pretty simple statement already but if you tell me which part you're struggling with I can explain it to you.

I've explained it to you. I don't know how much you know about SR, so I'm not sure where you want me to start helping you. It seems you'd rather just continue to make baseless claims, presumably because it makes you feel superior. It is clear you have little grasp on the topic, so instead of trying to discuss, you deflect and act like nothing is being said. It's bordering on being intellectually dishonest. Feel free to come back when you want to have an actual discussion.

And once again you just dodge the question, surprise surprise.

Okay then, I guess if I try to explain it in little baby steps for you, you might stand a chance.

Do you understand why flat earthers think the Earth can continuously accelerate but never reach the speed of light?

I can tell you are having a hard time, but please stop with your baseless claims. You didn't even ask me a question.

Explain what to me? I just pointed out your misunderstanding of how mass applies to SR in this particular case. But, please, feel free to continue being condescending to mask the fact that you really don't know what you are talking about.

I know why the flat earth can accelerate continuously and never reach the speed of light. I can't speak for others, nor will I attempt to.
Title: Re: You wouldn't know how fast you're going
Post by: TheTruthIsOnHere on April 12, 2016, 05:07:12 PM
No one is interested in an actual discussion. They just want to come here to try to force anyone with an opinion different then theirs to recant and bow to their infallible wisdom. There's obviously some kind of personal validation at stake and some kind of rigorous need to defend their position by attacking yours.

There's a lot of "thinking about what you're going to say next" while the other person is talking going on, and not a lot of active listening or understanding. I've withdrawn myself from this style of debate as it's pretty obvious who is dug into their position at all cost, usually just a glance at the username is enough to tell.
Title: Re: You wouldn't know how fast you're going
Post by: BCGreenwood on April 12, 2016, 05:12:14 PM
If you're disagreeing with what I'm saying I would suggest YOU actually read that same entry on special relativity.
So, you didn't read it, then. Gotcha.


Quote
If you still think I'm wrong I'm more than happy to talk about where our understanding of special relativity differs but obviously you'll have to be slightly more specific than "that isn't how that works".
You are talking about mass in absolute terms. Saying something like "nearing infinity" in this discussion is irrelevant. You obviously don't have a good enough understanding of SR to try and debunk UA with it. The model certainly isn't perfect, and there are other ways to scrutinize or criticize it. I suggest you look into those.


Quote
Just so we're clear, you're taking the stance that the Earth is travelling at 99.9% of the speed of light right now. And right now we're experiencing length contraction because of the speed we're currently travelling.
Again, this is a meaningless statement.


So again you can't actually say where I'm wrong, I'm sensing a pattern here where you just repeatedly dodge the question.

Just because you don't understand the statement doesn't make it meaningless. Well maybe meaningless to you because you don't understand it.

I can try and help you understand it if you'd like. It is a pretty simple statement already but if you tell me which part you're struggling with I can explain it to you.

I've explained it to you. I don't know how much you know about SR, so I'm not sure where you want me to start helping you. It seems you'd rather just continue to make baseless claims, presumably because it makes you feel superior. It is clear you have little grasp on the topic, so instead of trying to discuss, you deflect and act like nothing is being said. It's bordering on being intellectually dishonest. Feel free to come back when you want to have an actual discussion.

And once again you just dodge the question, surprise surprise.

Okay then, I guess if I try to explain it in little baby steps for you, you might stand a chance.

Do you understand why flat earthers think the Earth can continuously accelerate but never reach the speed of light?

I can tell you are having a hard time, but please stop with your baseless claims. You didn't even ask me a question.

Explain what to me? I just pointed out your misunderstanding of how mass applies to SR in this particular case. But, please, feel free to continue being condescending to mask the fact that you really don't know what you are talking about.

I know why the flat earth can accelerate continuously and never reach the speed of light. I can't speak for others, nor will I attempt to.

Excellent, you've finally answered something. Baby steps seems to be working for you, have a biscuit.

You're claiming you know why the flat earth can accelerate continuously and never reach the speed of light.
Do you understand that the flat earth is travelling at over 99.9% of the speed of light?
Title: Re: You wouldn't know how fast you're going
Post by: TheTruthIsOnHere on April 12, 2016, 05:14:20 PM
Excellent, you've finally answered something. Baby steps seems to be working for you, have a biscuit.

You're claiming you know why the flat earth can accelerate continuously and never reach the speed of light.
Do you understand that the flat earth is travelling at over 99.9% of the speed of light?

How do you know that the flat earth is traveling 99.9% of the speed of light.
How do you know how long the flat earth has been traveling?
How do you know the speed of light?
How does being a condescending prick work out for you in the real world bud?

There's some questions for you, try not to dodge them.
Title: Re: You wouldn't know how fast you're going
Post by: juner on April 12, 2016, 05:15:00 PM
No one is interested in an actual discussion. They just want to come here to try to force anyone with an opinion different then theirs to recant and bow to their infallible wisdom. There's obviously some kind of personal validation at stake and some kind of rigorous need to defend their position by attacking yours.

There's a lot of "thinking about what you're going to say next" while the other person is talking going on, and not a lot of active listening or understanding. I've withdrawn myself from this style of debate as it's pretty obvious who is dug into their position at all cost, usually just a glance at the username is enough to tell.

You are right. This newer batch of round earthers really don't seem genuinely interested in any honest discussion. They would rather parrot things they have been indoctrinated with to try to feel superior. I have seen a few of them spout off about how FErs don't understand math repeatedly. Turns out, they actually don't understand how the math works in their own model, but feel capable to criticize others.
Title: Re: You wouldn't know how fast you're going
Post by: juner on April 12, 2016, 05:19:38 PM
Excellent, you've finally answered something. Baby steps seems to be working for you, have a biscuit.
I really have no interest in continuing a discussion with someone who acts like a condescending prick. Especially when they don't understand the topic they are being condescending about. If at any point you want to have an actual discussion and can act like a reasonably mature adult, I will be around.
Title: Re: You wouldn't know how fast you're going
Post by: BCGreenwood on April 12, 2016, 05:27:44 PM
Excellent, you've finally answered something. Baby steps seems to be working for you, have a biscuit.
I really have no interest in continuing a discussion with someone who acts like a condescending prick. Especially when they don't understand the topic they are being condescending about. If at any point you want to have an actual discussion and can act like a reasonably mature adult, I will be around.

lol
I think you've worked out where this is going, congratulations. You've earned that biscuit, even though it means you believe in something stupid.

In simple terms flat earthers are trying to argue that the Earth is travelling close to the speed of light.
E=Mc2 so the mass of everything on Earth is ridiculously huge.

In even simpler terms flat earthers believe they have more mass in the palm of their hand than there is in a black hole.
Title: Re: You wouldn't know how fast you're going
Post by: TheTruthIsOnHere on April 12, 2016, 06:01:17 PM
Excellent, you've finally answered something. Baby steps seems to be working for you, have a biscuit.
I really have no interest in continuing a discussion with someone who acts like a condescending prick. Especially when they don't understand the topic they are being condescending about. If at any point you want to have an actual discussion and can act like a reasonably mature adult, I will be around.

lol
I think you've worked out where this is going, congratulations. You've earned that biscuit, even though it means you believe in something stupid.

In simple terms flat earthers are trying to argue that the Earth is travelling close to the speed of light.
E=Mc2 so the mass of everything on Earth is ridiculously huge.

In even simpler terms flat earthers believe they have more mass in the palm of their hand than there is in a black hole.

When you put it that way, special relativity sounds pretty fucking stupid. I know my hand existgs, but can you tell me when a black hole has every actually been proven to exist?
Title: Re: You wouldn't know how fast you're going
Post by: juner on April 12, 2016, 06:01:49 PM
Excellent, you've finally answered something. Baby steps seems to be working for you, have a biscuit.
I really have no interest in continuing a discussion with someone who acts like a condescending prick. Especially when they don't understand the topic they are being condescending about. If at any point you want to have an actual discussion and can act like a reasonably mature adult, I will be around.

lol
I think you've worked out where this is going, congratulations. You've earned that biscuit, even though it means you believe in something stupid.

In simple terms flat earthers are trying to argue that the Earth is travelling close to the speed of light.
E=Mc2 so the mass of everything on Earth is ridiculously huge.

In even simpler terms flat earthers believe they have more mass in the palm of their hand than there is in a black hole.

Again, that is not how it works. Inertial mass in acceleration within the context of SR refers to the energy required to keep accelerating an object, not that you literally gain mass. You are not applying E=mc2 properly. I would suggest you read the resource I provided as well as reading this:

http://physics.stackexchange.com/questions/91974/increase-in-mass-with-velocity

and this:
http://physics.stackexchange.com/questions/1686/why-does-the-relativistic-mass-of-an-object-increase-when-its-speed-approaches

You will see that there is not an absolute answer to the question at hand and you are grossly oversimplifying the situation.
Title: Re: You wouldn't know how fast you're going
Post by: BCGreenwood on April 12, 2016, 06:17:28 PM
Excellent, you've finally answered something. Baby steps seems to be working for you, have a biscuit.
I really have no interest in continuing a discussion with someone who acts like a condescending prick. Especially when they don't understand the topic they are being condescending about. If at any point you want to have an actual discussion and can act like a reasonably mature adult, I will be around.

lol
I think you've worked out where this is going, congratulations. You've earned that biscuit, even though it means you believe in something stupid.

In simple terms flat earthers are trying to argue that the Earth is travelling close to the speed of light.
E=Mc2 so the mass of everything on Earth is ridiculously huge.

In even simpler terms flat earthers believe they have more mass in the palm of their hand than there is in a black hole.

Again, that is not how it works. Inertial mass in acceleration within the context of SR refers to the energy required to keep accelerating an object, not that you literally gain mass. You are not applying E=mc2 properly. I would suggest you read the resource I provided as well as reading this:

http://physics.stackexchange.com/questions/91974/increase-in-mass-with-velocity

and this:
http://physics.stackexchange.com/questions/1686/why-does-the-relativistic-mass-of-an-object-increase-when-its-speed-approaches

You will see that there is not an absolute answer to the question at hand and you are grossly oversimplifying the situation.

Interesting that you direct me to a comments page.

"In special relativity, however, the inertial mass of a body directly depends on its speed - the higher the speed, the higher the mass. This effect is crucial for anyone operating a high-energy particle accelerator in which elementary particles are accelerated to speeds near that of light. "
http://www.einstein-online.info/elementary/specialRT/emc

That's from Einstein online if you want to read up on it.
Title: Re: You wouldn't know how fast you're going
Post by: juner on April 12, 2016, 06:44:35 PM
Interesting that you direct me to a comments page.
Yes, a discussion among people who research and study physics. I've provided you several resources thus far. You've added absolutely nothing to the conversation, other than repeating the same thing that has been demonstrated to be inaccurate.

Quote
"In special relativity, however, the inertial mass of a body directly depends on its speed - the higher the speed, the higher the mass. This effect is crucial for anyone operating a high-energy particle accelerator in which elementary particles are accelerated to speeds near that of light. "
http://www.einstein-online.info/elementary/specialRT/emc

That's from Einstein online if you want to read up on it.

Alright, I've given up on assuming you have any understanding of the concept. It is blatantly obvious that you do not. Let's focus on the quote you posted first:

Quote
"In special relativity, however, the inertial mass of a body directly depends on its speed

Looking more closely:
Quote
the inertial mass

As specific as it gets:
Quote
inertial mass

The link you posted literally says the same thing. I have to assume you are just trolling at this point. No one is this dense.
Title: Re: You wouldn't know how fast you're going
Post by: BCGreenwood on April 12, 2016, 07:19:02 PM
Interesting that you direct me to a comments page.
Yes, a discussion among people who research and study physics. I've provided you several resources thus far. You've added absolutely nothing to the conversation, other than repeating the same thing that has been demonstrated to be inaccurate.

Quote
"In special relativity, however, the inertial mass of a body directly depends on its speed - the higher the speed, the higher the mass. This effect is crucial for anyone operating a high-energy particle accelerator in which elementary particles are accelerated to speeds near that of light. "
http://www.einstein-online.info/elementary/specialRT/emc

That's from Einstein online if you want to read up on it.

Alright, I've given up on assuming you have any understanding of the concept. It is blatantly obvious that you do not. Let's focus on the quote you posted first:

Quote
"In special relativity, however, the inertial mass of a body directly depends on its speed

Looking more closely:
Quote
the inertial mass

As specific as it gets:
Quote
inertial mass

The link you posted literally says the same thing. I have to assume you are just trolling at this point. No one is this dense.

Interesting that you think the statement has been demonstrated to be inaccurate, the one written by the Max Planck Institute for gravitational physics.
But I shouldn't distract you, you seem to be a one thought at a time kinda person.

So simply my question is:
Does the intertial mass of a body increase as it's speed increases?
Title: Re: You wouldn't know how fast you're going
Post by: BlueMoon on April 12, 2016, 07:58:31 PM
Interesting that you direct me to a comments page.
Yes, a discussion among people who research and study physics. I've provided you several resources thus far. You've added absolutely nothing to the conversation, other than repeating the same thing that has been demonstrated to be inaccurate.

Quote
"In special relativity, however, the inertial mass of a body directly depends on its speed - the higher the speed, the higher the mass. This effect is crucial for anyone operating a high-energy particle accelerator in which elementary particles are accelerated to speeds near that of light. "
http://www.einstein-online.info/elementary/specialRT/emc (http://www.einstein-online.info/elementary/specialRT/emc)

That's from Einstein online if you want to read up on it.

Alright, I've given up on assuming you have any understanding of the concept. It is blatantly obvious that you do not. Let's focus on the quote you posted first:

Quote
"In special relativity, however, the inertial mass of a body directly depends on its speed

Looking more closely:
Quote
the inertial mass

As specific as it gets:
Quote
inertial mass

The link you posted literally says the same thing. I have to assume you are just trolling at this point. No one is this dense.

Interesting that you think the statement has been demonstrated to be inaccurate, the one written by the Max Planck Institute for gravitational physics.
But I shouldn't distract you, you seem to be a one thought at a time kinda person.

So simply my question is:
Does the intertial mass of a body increase as it's speed increases?


BCGreenwood, ease off a bit.  In this one narrow regard, they're right.  There is no absolute speed, so you could accelerate forever, and c would remain constant. 


Mind you, universal acceleration doesn't account for the measurable difference in gravitational acceleration at different latitudes (which we know is caused by the oblateness and spin of the earth) or what's keeping planets and satellites accelerating at nearly the same rate as everything else instead of falling to earth.  There are plenty of other areas where FEF falls apart, so let them have this one. 
Title: Re: You wouldn't know how fast you're going
Post by: TotesNotReptilian on April 12, 2016, 08:13:10 PM
As far as I'm concerned, this thread concluded with nametaken graciously accepting Rounder's explanation of relative motion in space (http://forum.tfes.org/index.php?topic=4764.msg92661#msg92661). But to clear up the argument that cropped up afterwards...

In simple terms flat earthers are trying to argue that the Earth is travelling close to the speed of light.
E=Mc2 so the mass of everything on Earth is ridiculously huge.

In even simpler terms flat earthers believe they have more mass in the palm of their hand than there is in a black hole.

If you want to talk about velocity, length contraction, and relativistic mass, you need to establish the frame of reference of the observer.

Let's call the observer Bob. Bob can survive in space. Bob is moving at .99 c relative to earth. Bob is pretty darn bored, because he doesn't get Netflix in space, but I digress...

According to Bob, the earth is indeed experiencing length contraction. The earth also has a ridiculously high relativistic mass.

According to the good citizens of earth, Bob is the one experiencing length contraction. Bob is the one with ridiculously high relativistic mass.

According to the good citizens of earth, the earth is not experiencing length contraction, and its total mass is just its normal everyday rest mass.

In conclusion, no one is implying that anyone is holding a black hole in their hand.

BCGreenwood: I don't mind people being wrong. I myself am wrong all the time. Heck, there is a chance I'm wrong about this. But being smug and insulting is a one-way ticket to no-respect land.
junker: You are generally correct about this topic as far as I can tell. But at least try to explain to the poor guy why he is being wrong, instead of being smug and dismissive about everything he says... its no wonder this thread was going nowhere.
Title: Re: You wouldn't know how fast you're going
Post by: Rounder on April 12, 2016, 08:21:05 PM
I suspect that theoretical physicists who truly understand the subjects of Special and General Relativity would get a good laugh at the attempts (both the RE and the FE attempts) to expound upon it here.
Title: Re: You wouldn't know how fast you're going
Post by: BCGreenwood on April 12, 2016, 08:29:04 PM
As far as I'm concerned, this thread concluded with nametaken graciously accepting Rounder's explanation of relative motion in space (http://forum.tfes.org/index.php?topic=4764.msg92661#msg92661). But to clear up the argument that cropped up afterwards...

In simple terms flat earthers are trying to argue that the Earth is travelling close to the speed of light.
E=Mc2 so the mass of everything on Earth is ridiculously huge.

In even simpler terms flat earthers believe they have more mass in the palm of their hand than there is in a black hole.

If you want to talk about velocity, length contraction, and relativistic mass, you need to establish the frame of reference of the observer.

Let's call the observer Bob. Bob can survive in space. Bob is moving at .99 c relative to earth. Bob is pretty darn bored, because he doesn't get Netflix in space, but I digress...

According to Bob, the earth is indeed experiencing length contraction. The earth also has a ridiculously high relativistic mass.

According to the good citizens of earth, Bob is the one experiencing length contraction. Bob is the one with ridiculously high relativistic mass.

According to the good citizens of earth, the earth is not experiencing length contraction, and its total mass is just its normal everyday rest mass.

In conclusion, no one is implying that anyone is holding a black hole in their hand.

BCGreenwood: I don't mind people being wrong. I myself am wrong all the time. Heck, there is a chance I'm wrong about this. But being smug and insulting is a one-way ticket to no-respect land.
junker: You are generally correct about this topic as far as I can tell. But at least try to explain to the poor guy why he is being wrong, instead of being smug and dismissive about everything he says... its no wonder this thread was going nowhere.


You've missed one important detail about the whole thing though, the twins paradox.
Yes Bob and Earth appear to each other to be moving very fast but only 1 of them is and that's the point I'm making.
Any observer would have to be travelling close to the speed of light for Earth to appear "normal".

Oh and Totesnotreptilian, I noticed you mentioned this on my other post I just couldn't be bothered to reply. You said 3 or 4 times that I didn't mention  that it was to an outside observer. To be honest, I thought it was kinda obvious.
Title: Re: You wouldn't know how fast you're going
Post by: TotesNotReptilian on April 12, 2016, 08:54:03 PM
You've missed one important detail about the whole thing though, the twins paradox.
Yes Bob and Earth appear to each other to be moving very fast but only 1 of them is and that's the point I'm making.

Either could be said to be moving, unless you accept the premise of an absolute frame of reference.

Unlike the twins' age, length contraction and relativistic mass are dependent only on the current relative velocity. They are independent of the path taken.

Quote
Any observer would have to be travelling close to the speed of light for Earth to appear "normal".

No, any observer would have to be travelling close to 0 m/s relative to the Earth for Earth to appear "normal". Forget about Bob. Bob's frame of reference is arbitrary and unimportant. If you want to measure the relativistic mass or length contraction of an object from our point of view, then the only frame of reference that matters is the Earth's.

Quote
You said 3 or 4 times that I didn't mention  that it was to an outside observer. To be honest, I thought it was kinda obvious.

My apologies. I like to be repetitive. I like to be repetitive. I like to be repetitive.

Title: Re: You wouldn't know how fast you're going
Post by: Rounder on April 12, 2016, 08:55:54 PM
Unless I've missed something, I really think all this discussion of relativistic effects is moot for the simple reason that if UA really is Universal, it means that all matter is subject to it, and there basically is no "outside" from which to observe, nor for us "inside" to make observations of.  Length contraction, increase in mass, all of it: those things only matter if there are two different frames of reference.  With nothing outside the accelerating frame of reference, we on the inside cannot say one way or the other if those effects are happening.  Going back to Einstein's thought experiments, his 'man in an elevator' subject could have been travelling at 99.9999c and subject to immense length and mass changes, but he himself would not know that unless he could observe something outside his own frame of reference and compare himself to it.
Title: Re: You wouldn't know how fast you're going
Post by: Round fact on April 12, 2016, 09:13:28 PM
Excellent, you've finally answered something. Baby steps seems to be working for you, have a biscuit.

You're claiming you know why the flat earth can accelerate continuously and never reach the speed of light.
Do you understand that the flat earth is travelling at over 99.9% of the speed of light?

How do you know that the flat earth is traveling 99.9% of the speed of light.
How do you know how long the flat earth has been traveling?
How do you know the speed of light?
How does being a condescending prick work out for you in the real world bud?

There's some questions for you, try not to dodge them.

Point 1. Math. If one is accelerating at 1g in order to maintain 1g you are mathematically required to gain speed. But other things are going during the acceleration. Too much so to go into detail here which has been proved by both sides; FE and RE alike.

Point 2. Pick a starting year. 4.8 billion years ago or 6020 years ago or any year you personally think the Earth was created, and plug in the numbers for 1g acceleration. The web is full of calculators that are a great help with doing the math for you. Or you can copy the formula and plug in the numbers yourself to get the answer.

Point 3. I am tempted to just respond with; Seriously? I honestly don't know where to begin. Radio, TV, Microwave, Cell phones, computers, wifi, everything electronic, you flashlight even. The speed of light is as well understood as basic checkbook accounting.

Point 4. Well you didn't address that to me specifically, and I won't answer for him except to say, I have seen the same behavior from you a few times on these boards. Not judging mind you, just pointing out an observation. 

Set in my ways? My will tell you, "HELL YEAH he is." But she'll also tell you that I have taken the time to learn enough to have that strong opinion. AND she'll tell you that when it is proved that my opinion is wrong, I'll swallow my pride and admit I was wrong.

That is why I trust the math and geometry. Each has had thousands of years of checks and proofs under their belts. It is also why I trust science, it changes when proofs are provided.  Einstein was castigated for GR and SR math. But when science started understanding and proofs were discovered it gained acceptance. And when another Einstein comes along, and there WILL be another Einstein, the castigation will happen again and as understanding and proofs are discovered it will be accepted. Live and learn is not a bad way to live, all you have to do is be willing to let go of you fantasies and accept what is plainly proved with math and science.
Title: Re: You wouldn't know how fast you're going
Post by: TotesNotReptilian on April 14, 2016, 04:28:09 PM
How do you know that the flat earth is traveling 99.9% of the speed of light.
How do you know how long the flat earth has been traveling?
How do you know the speed of light?
How does being a condescending prick work out for you in the real world bud?

There's some questions for you, try not to dodge them.

Point 1. Math. If one is accelerating at 1g in order to maintain 1g you are mathematically required to gain speed. But other things are going during the acceleration. Too much so to go into detail here which has been proved by both sides; FE and RE alike.

The whole point of the last few comments of this thread has been that claiming that "the earth is traveling at ____ speed" is completely pointless. You could declare the earth to be moving at absolutely any speed less than c, and be completely correct, depending on what you define as your frame of reference. (See Rounder's post. He gets it.)

Your other points stand.
Title: Re: You wouldn't know how fast you're going
Post by: Round fact on April 14, 2016, 04:40:17 PM
How do you know that the flat earth is traveling 99.9% of the speed of light.
How do you know how long the flat earth has been traveling?
How do you know the speed of light?
How does being a condescending prick work out for you in the real world bud?

There's some questions for you, try not to dodge them.

Point 1. Math. If one is accelerating at 1g in order to maintain 1g you are mathematically required to gain speed. But other things are going during the acceleration. Too much so to go into detail here which has been proved by both sides; FE and RE alike.

The whole point of the last few comments of this thread has been that claiming that "the earth is traveling at ____ speed" is completely pointless. You could declare the earth to be moving at absolutely any speed less than c, and be completely correct, depending on what you define as your frame of reference. (See Rounder's post. He gets it.)

Your other points stand.

Pease re-read my point again. You should find that we both agree with Rounder.
Title: Re: You wouldn't know how fast you're going
Post by: rabinoz on April 15, 2016, 03:05:45 AM
Unless I've missed something, I really think all this discussion of relativistic effects is moot for the simple reason that if UA really is Universal, it means that all matter is subject to it, and there basically is no "outside" from which to observe, nor for us "inside" to make observations of.  Length contraction, increase in mass, all of it: those things only matter if there are two different frames of reference.  With nothing outside the accelerating frame of reference, we on the inside cannot say one way or the other if those effects are happening.  Going back to Einstein's thought experiments, his 'man in an elevator' subject could have been travelling at 99.9999c and subject to immense length and mass changes, but he himself would not know that unless he could observe something outside his own frame of reference and compare himself to it.
I had not noticed your little 'man in an elevator' bit earlier.
One thing that seems to kill UA is that the 'elevator' is minute compared to the earth.

Einstein's Equivalence Principle is only valid for a region of space with a uniform acceleration due to gravitation.
This is clearly not satisfied over the whole earth. Even on the "Flat Earth" we know that apparent "g" varies quite significantly with latitude, altitude and the proximity to massive bodies (particularly high density ores).

On the Globe even the direction of "g" changes - it is always directed (almost) to the centre of the earth.
The reference http://Einstein Online, Equivalence Principle (http://www.einstein-online.info/spotlights/equivalence_principle) looks at these cases where these "tidal forces" become significant.

The "massive elevator" on the right illustrates this (though with the Globe). Objects in the elevator are subjected to gravitational forces in different directions and so we cannot apply Einstein's EP to the whole elevator.

Spacer(http://www.einstein-online.info/images/spotlights/equivalence_principleI/tidal_elevator.gif)
In the TFES "the Wiki" tries to explain this away with its "Celestial Gravitation",
Quote
Celestial Gravitation
Celestial Gravitation is a part of some Flat Earth models which involve an attraction by all objects of mass on earth to the heavenly bodies. This is not the same as Gravity, since Celestial Gravitation does not imply an attraction between objects of mass on Earth. Celestial Gravitation accounts for tides and other gravimetric anomalies across the Earth's plane.

There there seem numerous reasons to question this. Not the least is the illogicality of the tiny celestial objects having a gravitational effect on the objects on earth, yet the almost infinitely more massive earth  ??? has no effect  ???.

As far as I am concerned this completely invalidates Einstein's Equivalence Principle as a justification for "Universal Acceleration".